Вы находитесь на странице: 1из 88

1.

If f & g are functions of x such that g (x) = f(x) then,

f(x) dx = g(x) + c

d {g(x)+c} = f(x), where c is called the constant of integration

2.
(i)

Standard

n+1

(ax+ b)
(ax + b) n dx
a(n+1)

(ii)

dx
ax+

(iii)

(iv)

(v)

Fodxrmula:
+ c, n

1
= ln (ax + b) + c
ba
1
e
+c
a ax+b

e dx =
ax+b

1 a

a dx =

px+q

px+q

+ c; a > 0

p na

1
a

sin (ax + b) dx = cos (ax + b) + c

(vi)

+c
(vii)

(viii)
+c
(ix)
(x)

(xi)

1
a

cos (ax + b) dx =

sin (ax + b)

tan(ax + b) dx ln sec (ax + b) +


ac

1
a

cot(ax + b) dx =

ln sin(ax + b)

1
a

sec (ax + b) dx tan(ax + b) + c

1 cot(ax + b)+ c

cosec(ax + b) dx =
a

1
a

sec (ax + b). tan (ax + b) dxsec (ax + b) + c

=
(xii)

1 cosec (ax + b) + c

cosec (ax + b). cot (ax + b) dxa=

(xiii)

secx dx = ln (secx + tanx) +

OR

c
(xiv)

tan

x
cosec x dx = ln (cosecx cotx) + c OR ln
2

ln tan

4 +2

+c

(xv)

(xvi)

(xvii)

(xviii)

(xix)

d x = sin 1x
+c
a
a2 2x
1
dx
x
= tan
a 2 + x2 a
+c
1
a
dx
1
x
= sec1
+c
2
x x2a a
a
dx
x2 +a2

dx
x2 a2
dx

(xx)

(xxi)

xd x

(xxii)

= ln

+ x2 +a2 ]
x

+ x2
a2 ]
x

(xxiv)

OR

cosh x + c
1

xa + c
1
ln x+a
2a

a2
x
sin1 + c
2
a

x +
x +
2
a2
2
x

x2 +a 2 dx = x2 +a 2 + n a

+ c
a
2
2

x2 a dx =2
2

x2

eax. sin bx dx =

+ b2

a
a

eax. cos bx dx =

a 2+ b2

x +
x2
2
+ c

2
n a a
2

(a sin bx

Theorems

b cos bx) + c

(a cos bx + b sin bx) + c

integration

on

(i)

c f(x).dx

(ii)

f(x).dx
(f(x) g(x))dx =

g(x)dx

= c

x
+c
a

eax

3.

sinh1

eax
(xxvi)

OR

a+x
1
ln ax + c
x 2a

x
a2 x 2 dx = a 2 x2
2

(xxiii)

(xxv)

= ln

f(x)dx

(iii)
Note : (i)
(ii)

f(ax

f(x)dx = g(x)+

+ b)dx = g(ax + b)

+ c is integrable
c
every
contineous function
the integral of a function reffered onlyaby a
constant.
f(x).dx = g(x) +

= h(x) + c
c
&
g(x) = f(x)
h(x) = f(x)
g(x) h(x) = 0
means, g(x) h(x) = c

4x
5

Example :

Evaluate :

Solution.

4x

Example :

3
7 2
2
dx
Evaluate : x + 5x 4x +

x
+

Solution.

dx

4
2
dx = 6x6 + C = 3x6 + C.

7 2
2

dx
+ 5x 4x +
x

5x
2

= x dx +
=3+

dx

4dx x

dx +

2 dx
x

1
1/ 2

x
3
+2.
4x + 7 log | x | + 2
x1/2
= x
dx
+dx5 .+ 5
x
.
x
dx

4
.
1.
dx
+
7
.
+
C
3
4
x
4

4
5
x
+ x3 4x + 7 log | x | + 4
4
3 C

=
Example :
Solution.

Evaluate : exloga + ealogx + ealoga


dx
We have,

+ ealog x + ealoga
dx

xloga

dx

= exlogax a+ elogx
a + elogaa
= (a + x a+ a )
dx

= a dx
+
x
dx

x dx +a

ax
xa+1
= loga +
+ a . x + C.
a
a +1
2x + 3x
dx
5x

Example :

Evaluate :

Solution.

2 5+ 3 dx

2x

3x

5+ 5 dx
x

x +

dx

x1/


2 x

+ dx =


5 5

Example:

Evaluate :

sin

Solution.

1
= 8

(2sinxcosx)

1
= 8

sin

1
= 8

(2/5)x
(3/5)x
log 2/5 l+
og 3/5 + C
e

3
xcos
x dx
3 dx

2x dx

3sin2x sin6x
dx
4
(3sin2x
sin6x)

1
=
32

= 1
32

cos2x + 1 cos6x
dx
3

6
2
+ C

Example :

Evaluate :

Solution.

x4

dx
x +1
2

x4

dx
x +1

4
4
x 1+1dx = x 1+ 1 dx
2
2
2
x +1
x +1 x +1

2
= (x 1) dx
+

1
2

x +1

Example:

1
Evaluate : 4 + 9x2dx

Solution.

We have

dx =

x3
3

1
x + tan
x+C

4 + 9x

1
= 9

4 + x

dx

9
1
= 9

1
2

(2/3)

dx
+ x2

x

1
1
1
1

= 9 . (2/3)tan1
3x2
2/3 + C6 tan

+C
=
Example :

cosxcos2xdx

Solution.

cosxcos2xdx
=

1
2

2cosx

cos2x dx

1
2

(cos3x

+ cosx)

dxsin3x

=1

3
2
+ sinx

+c
1

Self Practice Problems


1.

Evaluate :

2.

Evaluate :

1+ sinx

4.

Integration
ions
If we subsitute
then
(i)
(ii)
(iii)

2
tan
x dx

Ans.

Ans.

dx

x = (t) in a integral
everywhere x will be replaced in terms of t.
dx also gets converted in terms of dt.
(t) should be able to take all possible value that x can take.
Evaluate :

Solution.

We have
=

L e t

3x

4
sinx
dx

=t

sinx4dx

d(x ) = dt
4

(n x)2
dx
x

Example :

Solution.

(n x)2 dx
x
nx = t
Put

1
dx = dt
x

dx
= t2 .
x

=
=

t dt
2

t3
3

+c

= (nx)3
3

Solution.

tanx sec x + C

by Subsitut

Example :

Example :

tanx x + C

+c

Evaluate (1+ sin x)cosxdx


Put
sinx = t
cosx dx = dt
t
(1+
+
2

) dt = t

t3
3 +c

1
4x3 dx = dt dx
4x3 dt
=

= sin x +sin x + c
3
Example :

Evaluate :

Solution.

We have,

x
4

dx

x + x +1

x
dx = (x2)2 + x2 +1dx
x + x +1
2

Let x = t, then, d 2(x


) = dt
2x dx = dt
=

x
dt

1
2

= 1
3

1
t +

3 2

2
+

1
t +
2 + C
3

tan1

3
2

2 2x

1
2t +1
1

tan
+1 +
3 + C
3

3
=
C.

tan

dx =
f(x)
n

[f (x)]

(f(x))1n

x(xdx+1)n

(iii)

(iv)

(v)

dt

f (x) dx = (f(x))n+1
n +1
[ f(x)]

(ii)

t2 + t +1 . 2x
1
t2 + t +1 dt

1
=
.
2

Note: (i)

1
=
2
=

dx
(n1)
n

x2(xn+1)

dx
1/n

xn (1 + x

Self Practice Problems

1n
N Take xn common & put 1 + xn =
t.

N, take nxcommon & put


1+x

= tn

x t.
take xn common as x and put 1 +n =

sec x dx

1.

1+ tanx

2.

sin(nx) dx
x

Ans.

n |1 + tan x| + C

Ans.

cos (n x) + C

dx =

dt
2x

Pa rt

5 .

Inte gration

by
:

f(x) g(x))
(f(x)
(i)
(ii)
(iii)

Example :
Solution.

dx =

(g(x)) dx

(f(x))
dx
g(x))dx
dx then it will not contain arbitarary
when you find integral g(x)dx

constant.
the choice of f(x) and g(x) is decided by ILATE rule.
g(x)dx
should
be taken
terms.
the
function
will come
lateras
is same
taken both
an integral
function.
Inverse function

L
Logrithimic function
A
Algeberic function

Trigonometric function
E
Exponential function

dx
xtanx
1

Evaluate :

xtanx
1dx

= (tan1

x) x2

1+ x

1
= x2
1
2 tan x 2

. x dx
2
2

x2 +11dx = x2
1
tan1 x
2
2
2
x +1

1x

1
+1dx

1
= x2
1
1
2 tan x 2 [x tan x] + C.

Example :
Solution.

dxx)
x log(1+
Evaluate :
dxlog (x + 1) . 2 x
=

= x2

2
=

log (x + 1) 1
2

1
xx +1

log (x + 1) x2 1+1 dx
x +1
2

+1
dx
x +1

1
1
dx
log (x + 1) (x 1)
2 +
x +1
2

x2

x2

log (x + 1) 1
x + log| x +1|
+C
2

= x2

Example :
Evaluate :

Let =
=

. x2

dx
2
1
x +1
x2 dx = x2
log (x + 1) 1
2 x +11
2

= x2

Solution.

x log(1+ x)

2x

2x

2x

sin3x dx

sin3x dx. Then,

sin3x dx

cos3x
= e2x 3

cos3x

dx
3
2
cos 3x + 3 e2x cos3x dx

1
= 3 e2x

1
2
= 3 e2x cos 3x + 3

e2x

2e

2x

sin3x 2e2xsin3x
3
3 dx

1
2
4
= 3 e2x cos 3x + 9 e2x sin 3x 9e2x sin3x dx

1
2
4
= 3 e2x cos 3x + 9 e2x sin 3x 9

e2x
4
+ 9 =9 (2 sin 3x 3 cos 3x)

e2x
13
=
(2 sin 3x 3 cos 3x)
9
9

e2x

(2 sin 3x 3 cos 3x) + C

13

Note :
(i)

[f(x) + f (x)] dx = e f(x) +


e
c

(ii)

x.

x
Example : [f(x) e+
dx = x f(x) +
(x)]2dx
x xf
(x +1)

c
Solution.

dx

Example :

Solution.

Solution.

x2

put

1cosec2 cot
x

dx
2
2

dx
2
(nx)
x=e

n (nx)

dx
e (x +1)
(x
+1)

(x +1)

ex 1 sinx dx
1 cosx

x cos x
1 2sin2

dx
ex
2sin2 x

Example :

x +11

nt + 1

dt t2

ex
(x +1) + c

x
= e cot + c
2
x


1 1 1 dt
nt + +
t t t2

1
n (nx)
x
+c
nx

1
= e nt +
c
t

Self Practice Problems


1.

2.

xsinxdx
2 x

e dx

Ans.

x cosx + sin x + C

Ans.

x e 2xe + 2e +
C
2

6.

Integration of Rational Algebraic Functions by using Partial


Fractions:

PARTIAL FRACTIONS :
f(x)
If f(x) and g(x) are two polynomials, then g(x) defines a rational algebraic function of a rational
function
of x.
f(x)
If degree of f(x) < degree of g(x), then g(x) is called a proper rational
function.
f(x)
If degree of f(x) degree of g(x) then g(x) is called an improper rational
function
f(x)
f(x)
If g(x) is an improper rational function, we divide f(x) by g(x) so that the rational
g(x) is
function
(x)
expressed in the form (x)g(x) where (x) and (x) are polynomials such that the degree of (x) is
+
f(x)
less than that of g(x). Thus, g(x) is expressible as the sum of a polynomial and a proper rational
function.
f(x)
Any proper rational function g(x) can be expressed as the sum of rational functions, each having a
simple factor of g(x). Each such fraction is called a partial fraction and the process of obtained them is
f(x)
called the resolutions or decomposition of g(x) into partial fractions.
f(x)
The resolution of g(x) into partial fractions depends mainly upon the nature of the factors of g(x) as
discussed below.
CASE I When denominator is expressible as the product of non-repeating linear factors.
Let g(x) = (x 1 a ) (x 2 a ) .....(xn a ). Then, we assume that
A1
A2
An
f(x)
=
x

a
1 + x a+
2 ..... +
g(x)
x an
where A1 , A , ...... nA are constants and can be determined by equating the numerator on R.H.S. to the
2

numerator on L.H.S. and then substituting 1x =


a , a , ........,a
2
n.
3x + 2

Example :

Resolve

Solution.

We have,

Let

into partial fractions.


2
x 6x
+11x
6
3

3x + 2

3x + 2
=
(x 1)(x 2)(x
x 6x +11x 3)
6
3x + 2
B
A
B
(x 1)(x 2)(x 3) = x 1 + x 2 + x 3 .
Then,
3

3x + 2
A(x 2)(x 3) +B(x 1)(x 3) + C(x 1)(x
(x 1)(x 2)(x 2)
(x 1)(x 2)(x
=
3)
3)

3x + 2 = A(x 2) (x 3) + B (x 1) (x 3) + C(x 1) (x...........(i)


2)
Putting x 1 = 0 or x = 1 in (i), we
get
5 = A(1 2) (1 3) A = 5
,
2

Putting x 2 = 0 or, x = 2 in (i), we obtain


8 = B (2 1) (2 3) B = 8.
Putting x 3 = 0 or, x = 3 in (i), we obtain
11 = C (3 1) (3 2) C = 11
2 .

3x + 2

3x + 2
5
11
8
= (x 1)(x 2)(x 3) = 2(x
+
2(x
3)
x 2
x 6x +11x 1)
6
3

Note : In order to determine the value of constants in the numerator of the partial fraction corresponding to
the
non-repeated linear factor px + q in the denominator of a rational expression, we may proceed as
follows :
q
Replace x = p (obtained by putting px + q = 0) everywhere in the given rational expression except
in
the factor px + q itself. For example, in the above illustration the value of A is obtained by replacing x
3x + 2
by 1 in all factors of
(x 1)(x 2)(x 3) except (x 1)
i.e.
31+ 2 5
A = (1 2)(1 = 2
3)
Similarly, we have
32 +1
33 + 2 11
B = (1 2)(2 3) = 8 and, C = (3 1)(3
=
2
2)
3

Example :
Solution.

Resolve x 2 6x +10x into partial fractions.


2
x 5x +
6 function is an improper rational function. On dividing we get
Here the given
3
2
(x + 4)
x 6x +10x = x 1 + (x2 5x +
2
2
6)
x 5x +
6

...........(i)

we have,

x + 4
2

x + 4
(x 2)(x 3)

x 5x +
6
x + 4
B
A
+
...........(ii)
So, let (x 2)(x 3)
x 2 x 3 x + 4 = A(x 3) + B(x
=
Putting x 3 = 0 or, x = 3 in2)(ii), we
get
B=
1. 2 in (ii), we
Putting1x = B(1)
2 = 0or,
x=
get
2 = A (2 3) A = 2
x + 4
1
2

(x 2)(x 3) = x 2 + x
3
3
2
2
2
Hence x 2 6x +10x = x 1
+
x 3
2
x

2
x 5x +
CASE II When the6denominator g(x) is expressible as the product of the linear factors such that some
of them are repeating.

1
1
Example g(x)= (x a)
k (x a )(x a ).......(x a ) this can be expressed
1
2
r
as
A3
A2
Ak
B1
B2
Br
A1
x a + (x + (x a)3+ ....+
+
(x a1) + (x a2) + ...... + (x ar
a)2
)
Now to determine constants we equate numerators on both sides. Some of the constants are
(x a)k
determined
by substitution as in case I and remaining are obtained by
The following example illustrate the procedure.
Example :
Solution.

3x 2
(3x 2)dx
Resolve (x 1)
(x 1) (x +1)(x
2 (x +1)(x + 2) into partial fractions, and evaluate
2
+ 2)
A2
3x 2
A3
A1
A4
Let
+
+
2
(x 1) (x +1)(x + 2) = x 1 + (x
x +1 x + 2
2 2 = A (x 1) (x + 1) (x + 2) + A (x + 1) (x + 2)
1)3x
1
2
2
+ A3 (x 1)
(x + 2) 4+ A (x 2 1) (x + 1) .......(i)
Putting x 1 = 0 or, x = 1 in (i) we
get
1
1 = A2 (1 + 1) (1 + 2)
A =
2
6
Putting x + 1 = 0 or, x = 1 in (i) we get

5 = A3 (2)2 (1 + 2) A3 = 5
4
Putting x + 2 = 0 or, x = 2 in (i) we
get
8
8 = A (3)
4

3
(1) of
A
= both sides, we get 0 = A + A + A4
Now equating coefficient
on
1
3
4
x
9
8
13
5

A1 = A3 A4 =
=
9
36
4

3x 2
1
13
5
8
(x 1)
2 (x +1)(x + 2) = 36(x 1) + 6(x

4(x
+1)
+
9(x
+ 2)
1)2

and hence

(3x 2)dx
2

(x 1) (x +1)(x +
2)
1
13
8
5
n |x 1|
=
n
|x
+
1|
+
n |x + 2| + c
6(x
1)

4
36
9

CASE III When some of the factors of denominator g(x) are quadratic but non-repeating. Corresponding
Ax +B
to each quadratic factor2ax
+ bx + c, we assume partial fraction of theax2 + bx + c , where A
type
and
B are constants to be determined by comparing coefficients of similar powers of x in the numerator of
A(2ax + b)
B
both sides. In practice it is advisable to assume partial fractions ofax
the
2 + bx + c + 2 ax + bx
type
+ c
The following example illustrates the procedure
Example :

2x 1
2x 1
Resolve (x +1)(x
+ 2) into partial fractions and evaluate 2 (x +1)(x
2
2) dx

Solution.

Let

2x 1
Bx + C
A
(x +1)(x
+ 2) = x +12 + x + 2 .
2
Then,

2
2x 1
A(x + 2) + (Bx + C)(x
2
2
(x +1)(x
+ 2) +1)
(x +1)(x +
= 1 = A (x + 2) +2)(Bx + C) (x +

2x
...(i)
1)
Putting x + 1 = 0 or, x = 1 in (i), we get 3 = A(3) A = 1.
2
Comparing coefficients of the like powers of x on both sides of (i), we get
A + B = 0, C + 2A = 1 and C + B = 2
1 + B = 0, C 2 = 1 (Putting A = 1)

B = 1, C = 1

2x 1
x +1
1
(x +1)(x
+ 2) = x +12 + x
2
+ 2

Hence

(x +1)(x

2x 1
2

dx

+ 2)

1
x
= n |x + 1| + 1 n |x2
+ 1| +
tan1
+c
2
2
2

CASE IV When some of the factors of the denominator g(x) are quadratic and repeating fractions of the

A2
A 0 (2ax + b)
A1 (2ax + b)
A1
+

form ax
+ bx +2c ax
++ 2
2
+2
)
2 (ax2 + bx + c)

ax
+
bx
+
bx + c

c
A2k1(2ax + b)

A2k

+
+ .......+
)k (ax2 + bx +)k
( 2
ax + bx + c
The following example illustrates the procedure.
c
2x 3
Example:
Resolve
into partial fractions.
2
(x 1)(x
+1)2
2x 3
Bx + C Dx +E
A
Solution.
Let (x 1)(x
=
+ 2
+ 2
. Then,
2
x 1 x +1
(x +1)2
+1)2
2
2x 3 = A(x
+ 1)2 + (Bx + C) (x 1) 2(x
+ 1) + (Dx + E) (x
......(i)
1)
1
Putting x = 1 in (i), we get 1 = A (12
+A =
4
1)

Equation coefficients of like powers of x, we have


A + B = 0, C B = 0, 2A + B C + D = 0, C + E B D = 2 and A C E = 3.
1
Putting A = and solving these equations, we get
4
1
1
5
B=
= C, D = and E =
2
2
4
2x 3

(x 1)(x
+1)2
Example :
Solution.

x +1
x+ 5
1
= 4(x 1) + 4(x
+1) +
2
2 2(x
+1)2

2x

Resolve

into partial fractions.


x 1
2x
2x
We have, 3
= (x 1)(x2 + x
x 1+1)
3

2x
So, let (x 1)(x
2
Then,

Bx + C
A
+ x +1) = x 1
2 + x + x +1 .

2
2x = A (x
+ x + 1) + (Bx + C) (x
.......(i)
1)
2
Putting x 1 = 0 or, x = 1 in (i), we get 2 = 3 A A =
3
2
Putting x = 0 in (i), we get A C = 0 C = A =
3
Putting x = 1 in (i), we get 2 = A + 2B 2 C.

2
3
2 = 3 + 2B 34 B =

2x
2
2x
2 1
2
2/3x + 2/3
1 x
1

+
or, 3
= 3.
= 3
+ 3 2
3
2
x
1
x
1
x 1
x + x +1
x 1
x + x +1

Self Practice Problems


1.

(i)

(ii)

(x + 2)(x +dx
3)

dx
2

(x +1)(x
+1)

7.
Integration
type

of

x+ 2
x + 3+ C

Ans.

Ans.

1 n |x + 1| 1 n (x2
1
+ 1) + tan1 (x) + C
2
4
2

+ + ,

+ +

+ +

dx

Express ax2 + bx + c in the form of perfect square & then apply the standard
results.

Example :

Evaluate :

Solution.

We have,

+ 2x + 5 dx

+ 2x +

dx

+ 2x +1+ 4

1 (x + 1)
= 22
2

(x 1)2 + 1
+ . (2)
2
2

log |(x + 1) + (x +1)2 + 22 | +


= 1 (x + 1) x2 2x + 5 + 2 log |(x + 1) 2 + x + 2x + 5 |
C
2
+ C
1
Evaluate : 2
dx
x x +1
1
dx
2
x x +1
1
=
dx
1 1 +1
2
x x4 4
+
1
=
dx
2
(x 1/2) + 3/
4
1
x 1/ 2 +
1
=
1
2
(x 1/2) + ( 3 /2)2 dx = 3 /2 C 3 /2

tan

2x 1
= 2 tan1
3
3 +
C.
1
Evaluate :
dx
9 + 8x x2
1
9 + 8x xdx
2

Example :

Solution.

Example :

Solution.

1
dx
{x 8x
9}
1
2

{x 8x +16
25}
1
{(x 24) dx =
52}

dx

4)2

x 4

dx = sin1
5
+C

(x

Self Practice Problems


1.
2.

8.

dx
2x + x 1

1
2

2x + 3x
2

dx

Integ ration of

+
+ +

dx,

Ans.

1n
3

Ans.

2x 1

2x +

log x

+C
2
3+ x + 3 x
+ 1 2
2
4

type
+

+ +

dx,

+ q)
(px
dx

2
ax
+ bx + c

Express px + q = A (differential coefficient of denominator) +


B.
Example :

Evaluate :

Solution.

2x + 3

2x + 3

dx
x + 4x +1
2

dx
x + 4x +1
2

+C

(2x + 4) 1
dx
2
x + 4x +1
2x + 4

dx
x + 4x +1
2

dt
t

x + 4x +1

1
2

dx

(x + 2) (3

dx, where t = x + 4x +
1
2

= 2 t log | (x + 2) + x + 4x +1 | +
C
= 2 x2 + 4x +1 log | x + 2 2 + x + 4x +1 |
+C

Evaluate : (x 5)2 x + x
dx
d
(x + x) + .
Let (x 5) = . dx
Then,
x 5 = (2x + 1) +2
Comparing
coefficients of like powers of x, we get
.

Example :
Solution.

1 = 2 and + = 5 = 1
2 and =
11
2
2
(x 5)x + x dx

1(2x
11
2

+1)
2
2 x + x dx
x2 + x dx11
2

11
+1) x2 + x dx 2

1(2x +1)

=1
2

(2x

+ x dx
+ x dx

1 2
1 2
+ dx where t =2 x
+x
2

2

1 1

x +
= 1 t3 / 2 11
2

x
+

.
2 2

1 2
1

2 3/2
2

2
2
1
2
1 1
x + + 1
1

. log
2
x + 2 2


2
2

+ C

1
2
1 3/2 11 2x +1 x2 + x
n
x
+

+
x
+
x

1
= 3t
8
2
2 4
+C

1
2

11
t dt 2

x +1 +
x + x
2x +1 x2 + x
1 n
1
2
11

+C
= 3(x2 + x)3/2
8
2

2 4

Self Practice Problems


1.

x +1
+x+3

dx

2.

6x 5

dx
3x 5x +1
2

Ans.
Ans.

2x +1
1
1
log |x + x + 3| 11 tan1
2 +
11 +

2
C
2

3x 5x +1 +
C

3.

1)
(x
dx

1+ x 2+ x

1 2
3
9
3/2
(x + x + 1)
8 (2x + 1) 1+ x + x2 log (2x +1 + 2 x2 + x +1 ) +
3
16 C

Ans.

9.

Int egrat ion of trigonometric

(i)

OR

OR

functions

Multiply Nr & Dr by sec x & put tan x = t.

(ii)

OR

OR

Hint:
Convert sines & cosines into their respective tangents of half the angles and then,
x
put tan 2 = t

(iii)

+
+

+
+ dx. Express Nr

d
A(Dr) + B (Dr) + c & proceed.
dx

Example :

1
Evaluate : 1+ sinx + cos x
dx

Solution.

1+ sinx
x

dx
+ cos
1

2
1 tan
x /2
1+
+
dx
2
2
1+ tan x /2 1+ tan x /
2

2tanx /2

1+ tan x /2

sec x /2 dx
dx =
2
+ 2tanx /2
1+ tan x / 2 + 2tanx /2 +1 tan x
/2
x
1
x
Putting tan = t and sec2
dx = dt, we get
2
2
2
=

dt = log | t + 1| + C = log
t +1tan

Example :

Evaluate :

Solution.

x +1 + C
2

3sinx + 2cos x

3cos x + 2sinx

dx2cos x
3sinx +

3cos x + 2sinx

dx
d
Let 3 sin x + 2 cos x = . (3 cos x + 2 sin x ) + (3 cos x + 2 sin x)
dx

3 sin x + 2 cos x = (3 sin x + 2 cos x) + (3 cos x + 2 sin x )


Comparing the coefficients of sin x and 12
cos x on both5sides, we get
3 + 2 = 3 and 2 + 3 = 2
= 13 and =
13

(3sinx + 2cosx) + (3cosx + 2sinx)


dx
3cos x + 2sinx

3sinx + 2cosx

x + 2sinx
1. dx + 3cos
dx

dt
= x + t , where t = 3 cos x + 2 sin x

= x + n | t | + C =
13
Example :

Evaluate :

Solution.

We have,
=

x+

12
n | 3 cos x + 2 sin x | + C
13

3cos x + 2
+3

sinx + 2cosx
dx

3cosx + 2
+3

sinx + 2cosx

Let 3dx
cos x + 2 = (sin x + 2 cos x + 3) + (cos x 2 sin x) +
Comparing the coefficients of sin x, cos x and constant term on both sides, we get
2 = 0, 2 + = 3, 3 + = 2

6
3
8
= 5 , 5 and =
5

(sinx + 2cosx + 3) + (cos x 2sinx) +


dx
sinx + 2cosx + 3

cosx 2sinx
1
sinx + 2cosx + 3
+ sinx + 2cosx + 3 dx +
dx

= x + log | sin x + 2 cos x + 3 | +


,
where
1
1
1 =
sinx + 2cosx + 3
dx
1 tan2 x / 2
2tanx /2
Putting, sin x =1+ tan
2 x /2 , cos x = 1+
2 tan x /2 we
get

dx

1 =

2tanx/2

dx
2
2(1 tan
x /2) +
+
3
2
2
1+ tan x /2 1+ tan x /2

tan xx/2/2 + 3(1+ tan x /2)


2tanx /2 + 21+
2tan
2
2
dx
2

sec x / 2

dx
2
tan x /2 + 2tanx /2
+ 5
x
1
x
Putting tan = t and sec2
= dt or
2
2
2
1 =

x
dx = 2 dt, we get
2

2dt
2

=2

sec2

+ 2t +
dt
2

(t +1) +
22

=
2
tan

x +1
tan

= tan
1
t +1
2

2
2
Hence, = x + log | sin x + 2 cos x + 3 | + 1
tan

x +1
tan

2 + C
2

6
3
8
where = 5 , = 5 and =
5
Example :

Solution.

dx
2

1+ 3cos x
2

sec x dx
2

tan x + 4

tanx
1
tan1 2 + C
2

Self Practice Problems


1.

4sinx + 5cos x
+ 4cos x
dx

5sinx

10.
Integration
type Case -

Ans.

of

40
9
x+
log |5sinx + 4cosx| + C
41
41

If m and n are even natural number then converts higher power into higher angles.
Case -
If at least m or n is odd natural number then if m is odd put cosx = t and vice-versa.
Case -
When m + n is a negative even integer then put tan x = t.
Example:

sin

Solution.

put

x cos4 x dx

Example :

(sinx)

1/ 3

Solution.

(sinx)

1/ 3

put

cos x = t

sinx dx = dt

(1 t

=
t

(t

=
dt

(t

t9 + 2t7
t5
5 +c
7
9

cos9 x
+2
9

2 2

) .t
4

. 4dt
2t +1)
dt
2

62t 4+ t )

Ans.
cos x cos x + c
5

(cosx)7 /3
dx

dx
(cosx)7 /3

1
(tanx)1/ 3cos x dx
2
tanx = t

3 4/3
t +c
4

t1/3dt

= 3 (tanx)4/3 + c
4

sec x dx = dt

Ans.

Example :

sin

Solution.

1
8

xcos x dx

sin

1
= 8
dx

2x(1+ cos2x)dx
1
2x dx + 8

sin
2

1
=
16

sin 2x cos2x

sin 2x
(1 cos4x) dx + 3

1
1
sin4x
sin3 2x
16
=

+
+c
64
48
16

11.
Integration
type

of

dx where K is any constant.

1
Divide Nr & Dr by x & put x = t.
x
Example :

Solution.

1 x2

dx
1+ x + x4
2

1
1
x2
dx
x2 + 1 +1
x2

1n
2

x+

1
=t
x

dt
1

t 1
+C
t +1
1
x + 1
x
+C
1
x + +1
x

1n
2

Example :

Evaluate :

Solution.

We have,
1
dx
=
4
x +1

x +1

dx

1
x2
2

1 dx

x +x2

1
=
2

2
x2
2

1 dx

x +x2

1+ 1
1 1
x2
x2

x2 + 1 x2 + 1 dx
x2
x2

1
=
2

1+ 1
1 1
x2
x2
1
1 dx 2
1 dx
2
2
x
2
x +
x +x2

= 1
2

1+ 1
x2

1 1
x2

dx
2

1
x
x +

1
1
Putting x = u in 1st integral and x += in 2nd integral, we get
x
x

1
=
2

1
2

1
dx
2
2
1
+ 2
x

du
+

2)2

2
(
2

)2

u 1 1
2
1
log + 2+ C
= 2 2 tan1

2
2 2 2

x 1/ x

x +1/ x 2
1
1
4 2 log x +1/ x + 2 +
= 2 2tan1

C
2
2
1 tan1
1
x 2 x +1
= 2 2
2 x 4 2 log
x + x 2 +1
2

+C

Self Practice Problem :

1.

2.

x 21

dx
x 7x +1

tanx

Ans.

Ans.

dx

1 2 . Integ ration of

Example:
Solution.

Evaluate :

OR

1n
6
1
2

1
x+
3
x
1
x+
3
xy

tan1

+C
+
1

y 2
1

+
2 2 n y + 2 + C where y = tan x tanx
2

type

; put px + q = t .
2

(x 3)

x +1
dx
1
Let = (x 3) x +1
dx
Here, P and Q both are linear, so we put Q = t

1
2

2t

(t 1 3)t2

dt

i.e. x + 1 = t and dx = 2t dt
2

t 2
dt
1
= 2 t2 22 = 2 . 2(2) log t ++ C
2

Example :

x +1 2
x +1 + 2 +
C.

1
log
2

x+ 2
Evaluate : (x2 + 3x + 3) x +1

dx

Solution.

x+ 2
Let = (x2 + 3x + 3) x +1

dx
Putting x + 1 =2,tand dx = 2t dt, we get

{(t
=

(t +1) 2t
2
dt
1)
+23(t 1) +

3} t2

=2

=2

1
where t t = u.
3)2

of
, put ax + b = ;

+1

1
t

+ +

1
t
t

3
+ C

2
3 (x +1)
3 tan1

+C

13.
Integration
type

Solution

du

= 2
3 tan1
t

Example :

u
2
2
= 3 tan1

3 + C =
tan1

1+ 1
2
t2
(t +1)
2
1
+1dt
t4 + t +1 dt = t2 +
t2
2

t 1
t

t2 t +1

1
t+

1 1
+1
t2 t

1 3
t +4
2

, put x =

2
1 + 3 +
1 +
t

= n t
2

2 C 4

(1+ x

Example :

) 1 x2

Solution.

Put x =

(t

t 1=y

put

1
2

+1) 2 t

y
1

= 2 tan1

2+ C

(y + 2) y

1
2 x
tan1
+

C
2x
Self Practice Problems :
1

dx

1.

(x + 2)

2.

Ans.

x +1

dx

Ans.

(x + 5x + 6) x
+1

3.

dx

(x +1)

4.

(2x

x2

5.

Ans.

1+ x

x2

dx

Ans.

+1) 1
dx

Ans.
2

(x + 2x + 2) x + 2x
4

1 4 . Integration

(
)

of

x +1) + C

x +1
x +1) 2 tan1
2 +
C

3
1
2x

sin1 +
1 5 + C

2 tan1

1
1

x
2
3 tan1
32
x +C

1
2 6n

x2 + 2x 4 6 (x
2
+C
+1)
x + 2x 4 + 6 (x
+1)

type

dx or

)(

) put x = cos2 + sin2

dx or

)(

) put x = sec2 tan

2 tan1

)(

put x
=t .

= t or x
2

1 5 . Red uction

rmula

fo

of

cosec

1.

2.

3.

=
n
tan

tan
n

n =

cot
n

x dx =

sec

sec
n

x tann2 x dx =2 (sec xn 1)
2
x dx

2
cot

cosec xcot
2

cotn1x

.cotn2 x dx =
n2

xdx
n2

n 2

n 1

x dx =

2
sec
x secn2 x dx

2 2) secn 3 x. secx tanx dx.

nn = tanx sec nn 22xxdx (tanx)(n


(n 2) (sec
x 1) sec n 2x dx
(n 1) n = tanx sec n 2x + (n n2) 2

tanxsecn2 x + n 2 n 2
n 1

n 1

cosec

2
n2
cosec
x cosec
x dx

dx =

n = cotx cosec

cotx cosec

n = cotx cosec

(n 1) = cotx cosec

n2

n2

x+

n =

n2

Solution.

cotxcosecn2x
2

(cotx)(n 2) ( cosec

x (n 2)

n2

n3

x cosec x cot x) dx

cot2 xcosecn2x dx

x (n 2)

Example :

2
(cosec
x 1)cotn2

n =

n =

sec

n 2

4.

cot x dx

n2

n =

xtann2 + dx

x dx =

n 1

2tan

tann1x

,
tan x dx

x dx

=
n
xdx

(cosec2x 1) cosecn 2 x dx

x + (n 2) 2

n2

+ n 2 n
n 1

n 1

n
Obtain reducation formula for
= x dx. Hence evaluate sin x dx
n

sin
n =

n 1
(sin x) (sin dx

x)
= cos x (sin x)
n1

= cos x (sin x)

+ (n 1)

n1
n1

(sin x)
(sin x)

n2

n2

cos2x dx

(1 sin 2x) dx

n = cos x (sin x)+ (n 1)


+ (n 1) n2 (n 1) n

x dx ,

n =

cosx(sinx)n1 (n 1)
+
n2
n
n

(n 2)

cosx(sinx)3
cosx(sinx)+ 1
3
x + C

Hence =
+
2
4

2
4
4
Self Practice Problems :
x 3
dx
x 4

1.

2.

[(x 1)(2 x)]

Ans.

dx

Ans.
3/

3.

[(x + 2)
]

4.

1/ 7

dx

(x 3)(x 4) + n ( x 3 + x 4)
+C
x 1 2 x

8 2 x x 1 +

6 1/ 7

(x 1)

Ans.

x 1
+C
7
2
x +

dx
Deduce the reduction formula for
(1+
= x4)n
n

Ans.

x
n = 4(n 1)(1+
x4)n1

4n 5
+ 4(n 1) n1

If

m
n
m,n = (sin x) (cos x) dx then prove that

m+1(cosx)n1
n 1
m,n = (sinx)
+
. m,n2
m+ n
m+ n

dx

and Hence evaluate =


(1+ x4)2

1
1

x
x +
x
3
1
1
tan1 x

n x

2 = 4 (1+ x4) 4 2 2
2 4 2
1

+
x

5.

+C

Definite Integrals
PART A :
A Let f(x) be a continuous
function defined on [a, b],
b

f(x)

dx = F(x) + c.

dx = F(b) F(a) is called definite integral. Thisformulais known


f(x)
asNewtona

Then
Note : Leibnitz formula.
1.

The indefinite integral


number.

2.
Given

f(x)

f(x) dx

is a function of x, where as definite integral f(x) dx is a


a

dx we can find
f(x) dx , but given
a

f(x) dx we cannot find

dx
2

dx
Illustration. 1 Evaluate (x +1)(x +
1
2)
1
1
1

Sol.
(x +1)(x + 2) = x +1 x (by partial
fractions)
+ 2

dx

(x +1)(x
1

2)

+
e (x +1)
e log (x
1
+ = [log
2)]2

4
= loge3 log
e
loge
Self Practice Problems

Evaluate the
following
2
5x2
1.
x2 + 4x + 3

2.

(2sec

x + x3 + 2) dx

Ans.

3.

Ans.

dx

1+ sec xdx

Ans.

2
3
log
+ elog
e

5
2

4
1024

= 8

9loge 4 log3 e
2

+2

3 3 + 2 log
e
3
18

PART B :
Properties of definite
integralb
b
P1
i.e.

f(x)
a

f(t) dt

definite integral is independent of variable of


integration.
b

P2

dx =

f(x) dx =

f(x)dx

f(x)

f(x)
b].

P3

dx =

f(x)dx +

f(x) dx, where c may lie inside or outside the interval [a,

5
x + 3 : x <
Illustration 2 If f(x) =
3
2
3x +1 : x 3, then f(x) dx
2
find

Sol.

=
f(x)dx
dx
2

f(x)dx +

f(x)

9 4
+ 3 (3 2) + 5
3 +53
2
3
3

2
(3x
+1)

(x + 3) dx +
dx
2
3

211
2

Illustration 3 Evaluate | x 5 | dx
2

Sol.

| x 5 | dx =
= 9
2
2

(x + 5) dx +

(x + 5) dx

Illustration 4 Show that (2x +1) dx (2x +1) +


+1)
=
0
5
0

(2x

L.H.S. = x + x ] 0 = 4 + 2 =
6 + 5 0 + (4 + 2) (25 + 5) =
R.H.S. = 25
2
2
6

L.H.S. R.H.S
Self Practice Problems
Sol.

Evaluate
thefollowing
2

|dxx

1.

+ 2x 3 |

Ans.

dx, where [x] is integral part of

Ans.

Ans.

13

0
3

2.

[x]
x.
0
9

[ t]

3.

dt

PART C :
a

P4
dx

f(x) dx =

(f(x) + f(x))

= 2 f(x) dx
even0
=0

if f(x) = f(x) i.e. f(x) is


if f(x) = f (x) i.e. f(x) is
odd

Illustration 5 Evaluate

1
ex + ex+ ex+ ex
ex + ex

dx =
1+ ex dx
1+ ex

0
1
1+ ex
x
1
e
+ ex x e (ex x+e ) 1 x
(e1 1) = 2 e
+
dx =
(e + ex) dx = e1

=
x
x
e
1
0 1+
e +1 1 +
0
e
1

Sol.

ex + ex
dx
1+ ex

cosx

Illustration 6 Evaluate

dx

Sol.

dx = 2
cosx
2

cosx dx =

( cos x is even function)

2x
Illustration 7 Evaluate loge

2+x
dx
1

2x

Sol.
Let f(x) = loge
2+x

2+x
2x
log
= log
= f(x)

f(x) =
e
e
2x

2+x
i.e.
f(x) is odd
function
1

2x
loge

dx = 0
2+x
1

Self Practice Problems

Evaluate
thefollowing
1
1.

| x | dx

Ans.

Ans.

Ans.

2.

sin

3.

x dx

cosx

1+ e

dx

PART D :
b

P5

f(x)
dx

dx =

Further

f(a + b x)

f(x) dx =
dx
0

f(a x)

Illustration 8 Prove that

g (sinx)

g (sinx) + g (cosx)dx
0

Sol.

g (cosx)

g (sinx) + g (cosx)dx = 4
0

g (sinx)

g (sinx) + g (cosx)dx

Let =

g
sin

x
2
2
g(cosx)

=
=
dx

g(cosx)+g(sinx)
0

0 g sin

x +
g
cos

x 2 2

on adding, we obtain

g (sinx)
+
2 =

g
(sinx)
+ g (cosx)
0
(sinx)
2

Note : 1.
2.

g (cosx)

dx =
g (cosx) + g

dx
0

The above illustration can be remembered as a


formula
Other similar formulae are

g (tanx)

g (cot x)

g (tanx) + g (cot dx = g (tanx) + g (cot dx = 4


0

x)

g (cosecx)

x)

g (sec x)

g (cosecx) + g (sec dx = g (cosecx) + g (sec dx = 4


0

x)

x)

g (x)

g (x) + g (a x) dxa2
0

Self Practice Problems


Evaluate
thefollowing

x
1.
1+ sinx dx

Ans.

Ans.

2 2 loge(1+ 2)

2.

sinx + cosxdx
0

3.

4.

dx
0 sin x + cos x

1+

PART E :

xsinx cosx
4

dx
tanx

Ans.

Ans.

2
16

12

=
4

2a

P6

f(x)

dx

= (f(x) + f(2a x))


dx
0

= 2 f(x) dx
f(x) 0
=0

if f (2a x) =
f(x)

if f (2a x) =

Illustration 9 Evaluate sin xcos x dx


0

Sol.

f( x) = f(x)
Let f(x) = sin x cos

x
3
3

sin xcos x dx =
0 0
3

dx
Illustration 10 Evaluate 1+ 2sin
2
x dx

Sol.

1
Let f(x) =
2
1+ 2sin x

f( x) = f(x)

dx
sec x dx
=2
=2
2
2
2
1+ 2sin x
0 1+ tan x + 2tan
0 1+ 2sin x

=2

sec x dx
1+
3tan x=
2

dx

2
3

[tan1(

2
tanx
0

)]

tan
is undefined, we take
2 limit

Lt
tan(1 3 tanx
) (tan1

x
2

3
) tan0

= 2
=
3 2
3
Note : We can evaluate the integral without using this
property

2
2
dx
cosec x
cosec x dx
Alternatively : 1+ 2sin
cosec
x
+
2
dx
=
2
2
2
x =
0
0
0
cot x +
Observe that we are not converting in terms3of tan x as it is not continuous in (0,
)

cotx

cotx

1
cot x
1 Lt tan1
Lt tan1
tan
1

3 =
x
3
3

3
3

x0
0

= 1
=

3 2 2 3

Note : If we convert in terms of tan x, then we have to break integral using property P
3.

Illustration 11 Prove that loge sinx dx =

0
0

log
cosx dx =
e

log (sin2x) dxlog


=e2
2
.

Sol.

Let =
dx
0

log
sinx
e

..........(i)

logsin
e

log

dx

(by property P 5)

..........(ii)

(cos x) dx

Adding (i) and (ii)

2=
log
e (sin x . cosx) dx =
loge 0
0

2=

log

(sin 2x) dx

2
e

log dx

2=

sin2x

2
dx

log
2

..........(iii)

where 1 =

log
e

(sin 2x) dx

put 2x = t

L.L:x=0

U.L:x=
2

1
dx = dt
2
t=0

t=

1 =

log

1
(sin t) dt
2

1
= 2
2

log

(sin t) dt (by using property P


6)

= log
2
2

1 =
(iii) gives

Self Practice Problems

Evaluate
thefollowing

logex +
1

x
dx
1+ x2
1.

2.

sin1x
x

dx

Ans :

loge
2

Ans :

loge2
2

xlog

3.

sinx dx

Ans :

2 loge
2

PART F :
P7
then

If f(x) is a periodic function with period T,


T

nT

f(x)

(i)

dx = f(x)dx, n z

n
0

a+nT

(ii)

f(x) dx = f(x)dx, n z, a
0R

nT

(iii)

f(x) dx = (n f(x)dx, m, n z

m)
mT

a+nT

(iv)

f(x) dx

=
nT

n z, a

b+nT

(v)

f(x)dx,
R
a

f(x) dx

=
a+nT

f(x)dx,
R

n z, a, b

Illustration 12 Evaluate

edx
{x}

1
2

Sol.

1+3

e{x} dx =

dx = 3 e{x} dx = 3
e{x}
1)
1
0
0

e{x} dx = 3(e

n+v

Illustration 13 Evaluate | cosx | dx , < v < and n


2 z
0

n+v

Sol.

n+v

| cosx | dx
| cosx |dx =| cos x | dx +

= cosx

cosx dx + n

| cos x | dx

= (1 0) (sin v 1) + 2n cosx dx
0

= 2 sin v + 2n (1 0) = 2n + 2 sin
v
Self Practice Problem
Evaluate
thefollowing
2

1.

e
1

{3x}

dx

Ans.

3 (e 1)

2000

2.

3.

dx
1+ esinx dx

5
4

sin2x

dx
sin x + cos x
4

Ans.

1000

Ans.

PART G :
P8
(x)

If (x) f(x)

for

then

a x b,

(x) dx
dx
a
a

f(x) dx

(x)

P9
a)

If m f(x) M for a x b, then m (b a) f(x)dx M (b


a

Further if f(x) is monotonically decreasing in (a, b) then f(b) (b a) <


if f(x)
a

f(x)dx < f(a) (b a) and

is monotonically increasing in (a, b) then f(a) (b a) < f(x) dx < f(b) (b


a)
a
b

P 10

f(x) dx

f(x)

dx

P 11 If f(x) 0 on [a, b] then f(x) dx


0
a
1
1
1
Illustration 14 For x (0, 1) arrange f (x) =
, f (x) =
and f (x) =
1
3
4 x2
4 x2 2
4 2x2
x3
1

order and hence prove that<6

Sol.

< 4 2

0
4x

0 < x3 < x2
x2 <xx32 + x3 < 2x2
2
4 2x2 < 4 x
3x < 4 x2

f1(x) dx <
0 dx
0
1

f3(x) dx <

<
6

f2(x)

x
sin <
2
0
0
1

dx
2

4x
x3
dx

4 x2 x3< 4 2

<

1
2

sin1

2x < x x < x
2

4 2x2< 4 2x x3 4 x2
for<x (0, 1)
f1(x) <3 f (x) <
f
2
(x)
1

dx

x 1

2
0

in ascending

sinx
Illustration 15 Estimate the value of
dx
x

sinx
Let f(x) =
x

Sol.

f(x) =

xcosx sinx (cos x)(x tanx)


=
<0
x2
x2
f(x) is monotonically decreasing
function.
f(0) is not defined, so we evaluate
Lt
Lt sinx = 1. Take f(0) =Lt f(x) = 1
x0+ f(x) =x0+
x0+
x
2

f =
2

2
0<
.
2

1<

sinx dx < 1 .

0x
0

sinx
dx <
x
2

Note : Here by making the use of graph we can make more appropriate approximation as in next
illustration.
1

x
Illustration 16 Estimate the value of
e dx using (i) rectangle, (ii)
triangle
0

Sol.

(i)

By using rectangle
1

x
Area OAED < e

dx < Area OABC

1<

x2

dx < 1 . e

1<

x2

dx < e

(ii) By using
triangle

x
Area OAED < e
dx < Area OAED + Area of triangle DEB
0

ex2 dx < 1 + 1
1<
. 1. (e 1)
2
0

1<

e
0

dx < e +1

x
x
Illustration 17 Estimate the value of
e dx by using
0

For x (0, 1), ex2 < e

Sol.

1<
1

11<
exdx 0

x2

ex2dx <
0

dx < e

Exercise : Prove the following :


1

1.

ex cos2 x dx <

2.

3.

0 < sinn+1 x dx <


dx 0
0

1
1
4 <

x3 cos x
1
1

2 + x2 dx <
2
2
0

1<

sin x

dx <

6.

dx < 1

5.

2 sin

x x

4.

2
ex2 cos
x dx

0<

x dx

16 + x < 16
0

PART - H
h(x)

Leibnitz Theorem :

If F(x) =

f(t)

g(x)

dt ,

then
dF(x)
= h(x) f(h(x)) g(x)
dx f(g(x))

e dx

Proof :

f(t)

Let P(t) =
dt

h(x)

f(t)

F(x) =

dt = P(h(x))

g(x)

P(g(x))
dF(x)
= P(h(x)) h(x) P(g(x)) g
dx (x)

= f(h(x)) h(x) f (g(x))g(x)


x2

Illustration 18 If F(x) = sint dt , then find F


(x)
x
Sol.

F(x) = 2x .

sinx2 1 .sinx

e3x

log

Illustration 19 If F(x) =

t
e

e2x

dt , then find first and second derivative of F(x) with respect to log
x
e

at x = loge
2

Sol.

e3x
e2x
dx
dF(x)
3 . e3x.
2 . e2x

x = e6x e .
) dx d (logx) =
logee3x
logee2x
4x

x
2 e

1
d
d
d F(x) =
6x
4x
d (logex) e ) = dx (e e ) dlogex = (6 e 4) ex
4x
d (logex (e6x
6x
4x

dF(x)
d(log

dx6 24 = 48
First derivative
of F(x) at x 2= log (i.e.
e
ex
2
2
x
6
4
2
6
2
Second derivative of F(x) at x =
(i.e.
e
=
2)
is
e
= 2) is (6
2 .2

4
.
2
)
.
log
e = 5 . 2 . log e.
log
x
2

et2 dt

Illustration 20 Evaluate 0
x
xLt
2
e2t dt

x
2

et2 dt

Lt 0

Sol.

2t

form

dt

Applying L Hospital
rule
x

x2

2 . et dt . e
=

Lt

1. e2x
x

2 . et dt
Lt

ex

Lt
x

2 . ex
2

2x . ex

=0

Modified Leibnitz Theorem :


h(x)

f(x,

If
F(x) =

g(x)
then
h(x)

(x) =

t) dt,

f(x, t)

g(x)

dt + f(x, h(x))h(x) f(x, g(x)) . g(x)

dt
Illustration 21 If f(x) = x + t , then find f
logex
(x)

Sol.

1
1
dt + 1 .
f(x) =

2
x
2x
logex (x + t)

1
2x

1
1
+
x + logex 2x

1
x (x +
logex

1
1
1
1

+
x x + logx
) =(x + t)
x
(x + xelog
(
2x
loge
e)

x +1
1
x (x + log
x
logx

x
dt
= log (x + t) (treating t as
Alternatively : f(x) =

e
x
+
t
constant)
logex
logex
x
f(x) = loge log (x + log
e
e)
2x
x
1
1
1
= loge 1
f(x) = (x + logex1+
) x x(x + log
x
e)
x

Definite Integrals dependent on parameters :


1

xb 1
, b being
x
e parameter

log

Illustration 23 Evaluate
0

Sol.

xb 1
Let (b) = logx dx

d (b)
db

xb logex

logex

dx + 0 0

(usingmodified Leibnitz
Theorem)
1
xb+11
1
b
=
= x dx =
b +10 b
0
+1
(b) = log (b + 1) +
c
b = 0 e
(0) = 0

c=0
(b) = log
(b+1)

Illustration 24 Evaluate tan (ax)dx , a being


x 1 x2 parameter
0

logxe 1
=
( x x+

)
e

(ax)
dx
tan
x 1 x

Sol.

Let (a) =

d (a)

1
x
dx
=
dx =
2 2
2
2
(1+ a x ) x 1
da
0
0
(1+ a x ) 1
x2
Put x = sin t
dx = cos t dt x2
L.L. : x = 0

t=0

U.L. : x = 1

d (a)
=
da

2
1
dt
cos
t
dt
=
2
2
cost
0 1+ a sin2 t
1+ a sin2 t

t=

sec t dt
2

1+ (1+ a )tan2
t
1

=
.
1+ a2 2
0

2
=
tan 1+ a tant

1+ a2
1
0

log
a + 1+a2 +

2 c e
But (0) = 0
c=0

(a) =

(a) =

log
a + 1+a2

2
e

Self Practice Problems :


x3

1.

If f(x) =
0

(x).
2.

t
If f(x) = e and g(x) = 1+ t4 dt then find the value of f
2
(2).
g(x)

3.

dt

1+ 4t

If x =

4.

Ans.

cos t dt, find f

d2y
and
dx2 = Ry then find R
2

3x2

cosx3

Ans.

17

Ans.

x2

If f(x) =
(x).
x

x sint
dt then find f
2

Ans.

2
sin x) + (cos x cos x )
x2 (2x sin x
x
2

5.

6.

If (x) = cos x
Ans.
0

cos x
(x t) (t) dt, then find the value of (x) + (x).

Find the value of the function f(x) = 1 + x te+2((log te) + 2log


1
vanishes.

2
dt where f(x) Ans. 1 + e

x2

cost

7.

8.

Evaluate Lt

xsinx

x0

dt
.

Ans.

Ans.

Evaluate log (1+ bcosx) dx, bbeing


parameter.
0
e

1+ 1b2
loge

PART -
Definite Integral as a Limit of Sum.
Let f(x) be a continuous real valued function defined on the closed interval [a, b] which is divided into n
parts
as shown in figure.
y = f(x)

a a+h a+2h ................ a+(n-1)h


a+nh=b

b a
The point of division on x-axis are a, a + h, a + 2h ..........a + (n 1)h, a + nh, = h.
n
where
Let Sn denotes the area of these n
rectangles.
Then, Sn = hf(a) + hf(a + h) + hf(a + 2h) + ........+hf(a + (n
1)h)
Clearly, S
is area very close to the area of the region bounded by curve y = f(x), xaxis and the
n
ordinates
x = a, x = b.
b

Hence

f(x) dx = Lt

Sn

n 1

f(x) dx = Lt

h f(a + rh) Lt

r=
= 0

b a
(b a) r

f a+
n

n
0

n1

n r=

Note :
1.
We can also write
n

b a
ba
n
a +
r
Sn = hf(a + h) + hf (a + 2h) + .........+ hf(a + nh) and f(x) dx =

n
f
n
Lt
r= 1
a

n 1
b

2.

If a = 0, b = 1,
=
0

Lt
f(x) dx
n

r=

1
f r
n
n

Steps to express the limit of sum as definte integral


r
1

Step 1. Replace by x, by dx andn Lt

n
n
by
Step 2. EvaluateLt
n

r
by putting least and greatest values of r as lower and upper limits
n
respectively.

pn

For examplen Lt

1fr
=
n n

r= 1

f(x) dx

r
Lt = 0,
nr =

n
1

Illustration 25 : Evaluate
1
1
1 +......... +
1
+
+

1+ n 2 + n 3 +
2n

Lt
n

Sol.

Lt
n

n
1
1
1 +......... +
1
+
+

1+ n 2 + n 3 +
2n
n

= n Lt

= Lt

1
r+ n

r= 1
n

r=

1
1
dx
1
n r
= x +1= [log
e (x +1)
0 ]
1
+1 0
2.
n

Illustration 26 : EvaluateLt

2n

Lt

Sol.

and

n+ r

2
r = 1n

n +1
n+2
3
2 +1+
n
2
n2 + 2+2
2n

= n Lt

+ r2

1
n

r= 1

n + +......... +
3

n2 + 32
5n

1+ r
n
2

r

1+
n

r

n
= 0, when r = 1, lower limit =
0

r
Lt = Lt
2n
n n
n
n
= 2, when r = 2n, upper limit =
2
2
2
2
1+ x
1
2x
1
dx =
dx +
dx
1+ x2
1+ x2
1+ x2
2
Lt

1
2 2
= tan x] 0+ 2loge (1+ x )

= tan 2 +
1

Illustration 27 :
Evaluate
Lt
n

1
log 5
2
e

n!
n
n n
1

Sol.

=
e log

Let y = Lt

n!
n
n n

n!
1
e
loge y = Ltn n log
n
n


r

n Lt
nr

= p)
= np


1. 2 . 3 ........n
1

= Lt
loge
nn
n n

1
2
3
= Lt 1 log
e + loge
+ loge +..... +

n
n
n n log
e
n n
n
1
= Lt
n n
=

r

log
n
e

r= 1

log x dx = xlog x

x
0
e
e

= (0 1) Lt
x0+

x log x + 0
e

= 1 0 = 1
1
y= e

Self Practice Problems :

1.

2.

Evaluate the following


limits
1
1
1
1
Lt
+
+
+.... +
2
2
2
n
n2
n + n n + 2n

2
n + n

Lt
n

n
3.

Lt

1
1
+
+

+
1
1+ n 2 + n 3+.......
+
5n
3
+ 2sin3 2 + 3sin3 3 +........ + nsin
1

n2
4n
4n
4n
sin3
4n
n 1

4.

Lt

5.

Lt
n

r= 0

PART J
Reduction Formulae in Definite Integrals

1.

If =
n

Proof : n =

n
n
sin
x dx , then show that =
n
1n

n
sin
x dx

n2

2 1

Ans.

loge 5

Ans.

2
(52 15)
92

Ans.

n2 r2

n
n
n
n
3
+
+
+......+
1+

n+ 3 n+ 6 n+ 9
n
n + 3(n 1)

Ans.

Ans.

x cosx]2 +
n = [ sinndx
1
0

(n 1) sinn2 x . cos2 x

= (n 1)
dx

2 sin x)
sinn2 x . (1

= (n 1)
1)
0

sinn2 x dx (n

sin

x dx

n + (n 1) = (n 1) n2
n

n 1
n2
n =
n

sin
dx

Note : 1.

n
cos
x

x dx =

n 1 n 3 n 5

0
n =
n n 2 n.....
or 1
4

according as n is even or odd.


=1 , =1
0
2
2.

n 1

n is even
1
if

nn
3 n2n
5

n
4
........

Hence = 22

n
n 1
........
. 2.1

if
n is odd

3
n

n n 2 n 4 3

2.

If n =
2

Sol.

n =

1
n
tan
x dx , then show that
=
+ n
n
n 1

(tanx)

2
. tan
x dx

n2

(tanx)

n2(sec

x 1) dx

=4

(tanx)

n2sec2x

dx

(tanx)
0

(tanx)n1
4
=
n2
n 1
0

n2
dx

1
n =
n2
n 1

n + n2 =

3.

If m,n =

sin
n

1
n 1

m 1
n
x . cos
x dx , then show that
= m + n m2 ,
m,n

Sol.

sin

m,n =

m1

x (sinx cos x) dx

2
x . cosn+1
cosn+1
x 2
m2
x
+
(m 1) sin x cos x dx
n +1
n
+1

0
0

=
m 1 2
n
2

n +1 sinm2 x . cos x . cos x dx

m 1 2
n
x m
sin x . ncos x
sinm2 x . cos
=
n +1 dx(
)

m 1
m 1
=


m2,n
n +1
n +1
m,n
m 1
m 1

1+
m,n
n +1
= n +1

m2,n

m 1
m,n =
n
m2,n
m+

Note : 1.

m 1 m 3

m 5
m,n =
m + n m + n 2
m+n
4
........ 0,n or 1,n according as m is even or odd.

0,n

=2

cos

x dx and 1,n
=

2.

Wallis Formula

sinx . cos
0

1
x dx =
n +1


(n 1) (n 3) (m 5).........(n 1) (n 3) (n
when both m,n are even
+ n) (m + n 2) (m + n
5)....... (m
2

4)........
m,n =

(m 1) (m 3) (m 5).........(n 1) (n 3) (n 5).......

otherwise

(m + n) (m + n 2) (m + n 4)........

sin

Illustration 28 :
Evaluate

x cos x(sinx + cosx)

dx

Sol.

3
2
Given integral = sin x cos x dx + sin x cos x dx

=0+2
dx
0

2
3
sin
x cos
x

( sin x cos x is odd and sin x cos x is even)


3

1. 2
4
= 2. 5 . 3 .1
=
15

5
6
xsin
xcos
x dx

Illustration 29 : Evaluate
0

Sol.

Let = xsin xcos x dx


0

= ( x) sin ( x) cos ( x)
0 dx

5
6
= sin . cos x dx
0

5
xsin
x . 6cos x dx

2 = . 2 sin x . cos x dx
0

4 . 2 . 5 . 3 .1
= 11. 9 . 7 . 5 . 3 .
1
8
=
693
1

Illustration 30 : Evaluatex (1 x) dx
0

Sol.

Put x = sin

:x=0
L.L

U.L.

:x=1

dx = 2 sin cos
d
=0

=
2

xd(1 x)
3

=2.
d

dx =

2
5
sin (cos
) 2 . sin . cos

7
sin
cos11

6 . 4 . 2 .10 . 8 . 6 . 4 . 2
1
= 2 . 18 .16 .14 .12 .10 . 8 . 6 . 4 . =
504
2
Self Practice Problems:
Evaluate
thefollowing

1.

sin

Ans.

x dx

2.

sin

Ans.

x cos x dx

3.

sin1 x dx

x (a2 x2)2
dx

16

14 245

Ans.

a9
9

5.

3/2

2 x dx

8
315

Ans.

4.

18
15

Ans.

KEY CONCEPTS
1.

DEFINITION :
If f & g are functions of x such that g(x) = f(x) then the function g is called a
PRIMITIVE OR
ANTIDERIVATIVE OR INTEGRAL of f(x) w.r.t. x and is written symbolically as
+ c} = f(x), where c is called the constant of
f(x) dx = g(x)+ c{g(x)

integration.
STANDARD
RESULTS :
d

2.
(i)
=

dx

(ax+ b)n+1

(ax + b)n dx

(iii) eax+b
eax+b + c

+c n
+ )1

(ii)

dx1 = a n 1
a

(iv)

(v) sin (ax+ b) dx =


a
b) + c
1
a =
(vii)
tan(ax+ b) dx
1
b) + c

cos (ax+

ln sec (ax+
tan(ax +

dx

= ln (ax + b) +
ac
+

ax b

1 a

apx+q dx

q
px+

p na

(a > 0) +
c

1
a
(vi)
cos(ax+ b) dx =
sin (ax+
b) + c
1
(viii)
cot(ax+b)dx ln sin(ax +b)+
ac

=
1
(ix) sec (ax + b) dx =
(x) cosec(ax + b) dx cot(ax + b)+
b) + c
ac
=
a
1
(xi) sec (ax + b) . tan (ax + b) dx =
sec (ax + b) +
c
a
(xii)
cosec (ax + b) . cot (ax + b) dx = 1 cosec
b) +
(xiii)(ax
+
secx
dxc= ln (secx +

OR

x
ln tan +
+c
4

tanx) + c
x
(xiv)
+ c OR ln (cosecx +
cosec x dx = ln (cosecx OR ln tan
2
cotx)
cotx) + c
(xv) sinh x dx = cosh x + c (xvi) cosh x dx = sinh x + c (xvii) sechx
dx= cosechx
tanh x + dx
c =
(xviii)

(xix) sech x . tanh x dx =

x + cx . coth x dx =
(xx) coth
cosech

x
x + c= sin1
(xxi) sech
+c
dx
a
a 2x

cosech x + c
dx = 1
x
(xxii)
+c
a 2+x tan
2 1
a
a
(xxiv) d x
= ln
x + x 2

x 2+ 2a +a

dx

(xxv)

x 2a
2

x+
= ln

dx = 1 a+ x
ln a x + c
a 2x 2 2a

dx = x
+ 2a
(xxviii)
a 2 x 2
2
sin1
a 2 x 2
2

(xxvi)

(xxiii)

dx

=1
a
x x 2a

secx1+ c
a

OR

sinh1x + c
a

OR

cosh1x
+c
a

(xxvii)
x
+c
a

= 1

2a ln

dx
x 2a

xa
+

x a
2

+c

+ a2
dx = x
2
x
2
+a
2
sinh
1
2
2
2
dx = x
a
x 2 a
x
2
a
2
cosh21
2
2

(xxix) x 2+a
(xxx)

(xxxi) eax. sin bx e

ax

a 2 +b

dx =
e
(xxxii) eax . cos bx
3.
(i)

(ii)

(iii)
4.
(i)
(ii)

x
+c
a
x
+c
a

(a sin bx b cos bx)


2
+
c

ax

(a cos bx + b sin bx)

a 2 +b +
2 c

dx =
TECHNIQUES
OF INTEGRATION :
Substitution or change of independent
variable .
f( (t)) f(t) dt , by a suitable
Integral I = f(x) dx is
substitution
x
= (t) provided
the later integral
is easier to
changed
to
integrate .

v du . vdx dx where u & v are


Integration by u.v dx =u
part :
dx differentiable

dx
function . Note : While using integration by parts, choose u &
v such that

(a) v dx is simple
(b) du . vd x dx
is simple to
d x integrate.

& is generally obtained, by keeping the order of u & v as per the order of the
This
letters in ILATE,
where ; IInverse
LLogarithmic
function
AAlgebraic
function,function,
TTrigonometric
function
& ,
EExponential function
Partial
fraction
spiliting
INTEGRALS
OF, THE
TYPEa :bigger fraction into smaller fraction by
known methods .
f (x)
OR
[ f(x)]n f(x)
[f(x)]n dx put f(x) = t &
proceed .
dx
dx
dx
ax+2 bx c+ , 2
, ax2 + bx + dx
c

ax + bx +

(iii)

(iv)
(vi)

Express ax2 + bx + c incthe form of perfect square & then apply the
standard
. px +
px + results
q
dx .
ax + bx +dx ,
q
+

c
bx + c
Express
px + q =ax
A 2(differential
co-efficient of
denominator)
+
B
.
2
(v)
ex [f(x) + f(x)] dxx = e .
[f(x) + xf(x)] dx = x
f(x)dx+ c

f(x)
++
cx
n N Take xn common & put
1

x(x n +1)
n = t .

(vii)

(viii)

(ix)

dx
x2(xn +1) n
(n1)n

n N ,
= tn

take xn common as x and put


1+x
=t.

dx
xn 1dx
+ xn)1/n
a+ bsin x2 OR
r

.
Multiply .N
=t .

take xn common & put 1+x

& . .D

dx
a+bcos
x
2

OR

dx

by sec x & put tan x

asin
ccos

2
2

+
x + bsin x cos x

dx
a+bsin

(x)

OR

dx

dx
a + b sin
x + c cos

OR
a+bcos x
x
x
Hint :Convert sines & cosines into their respective tangents of half the angles
, put tan
=t
2
a.cos x + b.sin cx
d
(xi)
(Dr) + c &
dx
.
Express
Nr

A(Dr)
+
+B
dx proceed .
.cosx x
+
m
.sin
x
+
2 1
+
x 2
(xii) n
dx OR
dx where K is any
1
x 4 + K x 21
constant .
x 4 x
+ K&
x 2 +
+ : Divide Nr & Dr by
Hint
1
proceed .
dx
(xiii) (ax +
&
q ; put px + q = t2 .
px + q
(ax2+ bx c+)
b)dx
px +
1
1
dx
dx
, put x =
, put ax + b =
(xiv) (ax + 2
t
t
(ax2 + bx + cpx
) 2 + qx +
px + qx +;
b)
r
r
x

(xv)
dx or x ( x) (;
x
put x = cos2 +
)
sin2
x

tan2
x dx or x ( x ) (

) ;
put x = sec2
dx
(x ) ( ; put x = t2 or x
x
= t2 .

DEFINITE

INTEGRAL

1.

2.

f(x) dx = F(b) F(a) where


f(x) dx =
b
F(x) + c

VERY IMPORTANT NOTE :


If a f(x) dx = 0 then the equation f(x) = 0 has
root lying in (a , b) provided f is a continuous function
atleast one
in (a , b) .
PROPERTIES OF DEFINITE INTEGRAL :

f(x) dx =

P1

same

f(t) dt provided f is

P2

f(x) dx = f(x)
a
b
dx

P3 f(x) dx
dx + f(x) dx , where c may lie inside or outside the interval [a,
= f(x)
a
a
c
b] . This property
to be used when f is piecewise continuous in
(a,
b) .
a
P4

f(x) dx = 0

if f(x) is an odd function i.e. f(x) =

f(x) .
a

P5

= 2 f(x) dx
0
= f(x)
.
b

f(x) dx =

f(a x)dx
a

if f(x) is an even function i.e. f(x)


a

f(a + b x) dx , In particular

f(x) dx =

2a

P6

f(x) dx =

f(x) dx
+

f(2a x) dx = 2

f(x) dx

if f(2a

x) = f(x)
=0
if f(2a x) =
na
f(x) a
P7 f(x) dx = n
f(x) dx ; whereais the period of the function i.e.
0 f(x)
0 f(a + x) =
0

P8
P9

b+ nT

a+ nT

where f(x) is periodic with period T &


f(x) dx = f(x)
dx
n
I.

na

dx = (n f(x) dx if f(x) is periodic with


f(x)
m)
0 period 'a' .
ma

b
b
P10 If f(x) (x) for a x b then f(x) dx
a
a
(x) dx
P11
dx .

b
a

f (x )dx

f(x)

P12 If f(x) 0 on the interval [a,


b] , then
a
f(x) dx 0.
3.
WALLIS FORMULA :
/2

4.

[ =
(n 1)(n
3)(n ][
1or
sinnx . cosmx dx
5)....
2 (m 1)(m
(m + n )(m + n 2)(m + n 4)....
0
1or
2
]
1or 2

K
Where K =
if both m and n are even (m,
;
2
n N)
=1
DERIVATIVE otherwise
OF ANTIDERIVATIVE FUNCTION :
If h(x) & g(x) are differentiable functions of x then ,

3)....

h(x)

5.

d f(t) dt = f [h (x)] . h(x) f [g (x)]


. g(x)
dx g(x)

DEFINITE INTEGRAL AS LIMIT OF A SUM :


b

Limit
f(x) dx =
n
a

)+ f a
h [f (a) + f (a + h) + f (a + 2h)( ++.....
]

n 1h

n1

h f (a + rh) where b a
=Limit
h0
=0
= rnh
n1

If a = 0 & b = 1 then , Limitn h f (rh) =


r=0
0
where nh = 1

f(x) dx ;OR

1 n1 r
Limit
f = f(x) dx
n
n r=1 n .

6.

ESTIMATION OF DEFINITE INTEGRAL :


b

(i)
(ii)

For a monotonic decreasing function in (a , b) ;a f(b).(b a) <


b
f(a).(b a) &

f(x) dx <

For a monotonic increasing function in (a , b) ;a f(a).(b a) <


< f(b).(b a)

f(x) dx

7.

SOME IMPORTANT EXPANSIONS :

(i)

1 1+ 1 1 ++..... = ln
213 4 5
2

(ii)

(iii)

1 1 + 1 1 +..... = 2
1 2 2 2 32 4 2
12

(iv)

1
1
+
22 4

(v)

Q.1
Q.4

2
6

1 +
3 2 14

+
+.....
1 + 1+
1 + 1 +..... = 2

=
1 2 32
7 2
2
8
5

EXERCISE1

d
cos6 + 6sin

dx

(x4

2
1
+ 1 +.....
6=
2
8 2
24

tan 2

1 +1
12 2

Q.2

4
+ 4 x5
5x
dx
(x + x +)1

Q.5 Integrate

dx

cos x
Q.3 1+ tan
2
x
dx

2 +2x1
by the substitution z x=
x+

x x + 2x1

2
x x1 e
cos +sin d
dx

Q.6 x

Q.8

sin
cos

sin
x

x
+sin 2x
edx

nx

Q.7
cos
2
.
ln

2
2
a2 sin2 +
x 2
b cos
dx
x
a
sin
x
b
cos
x
Q.10
2+
Q.9 4 2 + 4 2
Q.11 x+
(x +x(1 x))2
dx
dx
+
2
cot x dx x
sin(x a)dx

Q.13 (sinx) 11/3 (cos


x)
Q.12

(1sin x )(sec
sin(x+a)
1/3dx
Q.14 x+1)
x
x dx
dx
Q.15

a+ x
1+ x
Q.16 sin1
1
x 2+1
[ ln( x 2+1 ) 2ln
] dx Q.18ln(ln x )+ 1
x+1
Q.17
Q.19
dx

2
x 2
x
(ln x ) dx

x(1+xe
)

x 4

1
1 tan
x ln
Q.20 Integrate
f (x) w.r.t. x4 , where1 f (x) =
2
+
1 x
x + ln
( x +1)dx
x2 + x
dx
Q.21
Q.22
(e x 1)2dx
3
sin x2 cos3
Q.23 x + +
x ( x +1)
x
2
cosecxcotx secx
cosx sinx
dx
dx
Q.24 cosecx+cotx1+2secxdx Q.25 79sin2xdx
Q.26 secx cos ecx
.

dx
Q.27 sinx+secx

Q.28 tan x.tan 2x.tan


Q.29
3x dx
x2
3+4sinx+2cosx
Q.30(xcosx sin x)(xsin x + cosx)
dx sinx
Q.31
3+2sinx+cosx
dx
Q.32
Q.33

n (cosx+

sin x

dx

(
+ )
sin 2x

cos2x
)

dx

sinx
dx
sinx + cosx

Q.34

dx
sin x + tan
x

3x2 +1
dx
Q.35 2
(x 1)3

cosx

(xsin3 x +
e cosx)2
sin x

Q.36

Q.37

dx

Q.39 (7x 10 2 x )3dx


/
2

2 3x
1+dx
Q.42 x
2 + 3x
1x
x +
2
dx
x 2 +3x +3
)
Q.45 x+1
(

Q.43

Q.48

x (ax2 + b)2

x 2(1

xlnx3

/2

cot x

x2

Q.1

9cos x + sin
/2 x
0

Q.2

z
2

dx

1x

1x dx
1+x x
2

Q.50

(0, )

1 2x2 cos +
x4
e

1 sin2x
dx
1+ sin2x

(1x)

dx
(x
)(x
)
Q.47
)
(x
(1+ x2)dx

Q.49 sin x dx
3
2
x4 + 4x
6x
+ 4x
cos2x
+1

x dx

2
(2x
)

2 dx
x

EXERCISE2

4x
dx Q.444x 7x2 +2 8x 2x +dx
x (x +1)2
7

x2

xdx

Q.413

dx

tanx

1 + 3sin2x

Q.46

Q.38

2 2

x c

Q.40

(ax2 b) dx

Q.3 Evaluate
I =
n
find I3.

/2

(lnn x) dx hence

/4

x dx
Q.4 sin2x arc tan(sinx) dx
Q.5 cos4 3x . Q.6 cosx (cosx
+ sinx)
0
sin2 06x dx
0
d
sin x
(h(x)) =
Q.7 Let h (x) = (fog) (x) + K where K is any
then compute
dx
the
constant. If
cos2(cos x)
f (x)
f (t)
dt , where f and g are trigonometric
value of j (0) where j
g(t)functions.
(x) =

g(x)

Q.8

Find the value of the definite 2 sin x + 2cos x


integral
0 dx .
5

Evaluate the integral x: + 2 2x 4 + x 2 2x 4

3
dx

x2
xdx
dx
dx
;
Q
=
Q.10 If P =
and
R
1+ x
1 x4
= 1+ x 4 then prove
0
0
0

4
+
that
(a)
Q = , (b) P = R,
(c) P2 Q + R = 2
4
2
b xn1 n
2
x
2
+
(n
1)(a
+nab

bn1
Q.11 Prove thatb)x +
dx =n1
a
(x + a)2(x +
a
a b)
2( +
2
b)
2
1
4 (1
4
1
x)
x2 x dx
2
x
dx
x
.ln
x
Q.12 1+2x
Q.13
dx
Q.14 Evaluate:

x2 +
0
0
4
1 x 2
2
Q.9

sin1 2x dx
Q.15
1+ 2x
0
2

x
ex cos
dx
+
Q.18
4 2
0

eb g

/2

Q.16

asin x + bcos
+)
4 x

x sin(
0

Q.19

dx

Q.17

6 10x
5
+
2x7+3x
7x 2+12x
1dx
32
x2 +

dx
2+sin2x

xsin
Q.20 Let , be the distinct positive roots of the equation tan x = 2x
then x)dx
,

evaluate (sin
0
independent of

/4
and
. sin x
(ax b)secxtanx
cos x
(2x+ 3)sin x
Q.21
dx
Q.22 +

2
dx
dx
(a,b>0)
Q.23
4 +tan x
0
+
0
0
10 +sin 2x
2
Evaluate:
satisfy
Q.24 If a2, a and
a are the three values of
a which
3
(1
cos
x)
the equation1
1
1
4a
3
(sin x a cos x) dx 2x2 cos x dx =
0
0
then find the+value of
a2 + 2
a ).
p+q
1000(a12
+

2
3
p <
Q.25 Show that
| cos x| dx = 2q + sinp where q2 <
2
0 N &
5

2/3

dx + 3
Q.26 Show that the sum of the twointegrals e(x+5)
e9(x 2/3)
dx is zero.
1/3
4
1 sin 1 x dx
/2
1
x3
sin 2
1 +
x
n
x
Q.27 2 +
Q.28 2
dx
(a>0,
Q.29

2
x 1 x2dx
1
x
x 1
1
0 a sin 2 x+b
0x
b>0)

cos
/2

a2+b2
2

x + 1sin
Q.30 tan11+sin
1+sin x 1sin
x

x dx

Q.31

3a +b2
2

x.dx
(x2 a2)(b2
x2 )
1

Q.32 Comment upon the nature of roots of the quadratic equation


x2 + 2x = k + | t
+
dt depending
on
the
0
value of k +
2aR.

1
1 2a x
dx
dx
Q.33 x sin
1

(n > 1)
a dx
Q.34 Prove that1 + xn=
2

z2
Q.35 Show that
eex2 ez2 dz = 4e 4 dz
0
0
zx

dx

Q.37 (a) 1+
x . x+ x
0 1 x
+x

1+ 5
2

, (b)

Q.39

Q.36
0

x 2 sin 2x.sin
(2
x
2x

.cos
)

x2 +1
1
ln1 +x dx
x

x4 x2 +1
sin

(0,)

8+ sin2
0

(1 x n)1/n

dx

if
L
M
dx
dx
=M
Show thatx + 2x cos = 2 x
1
+ 2x cos + M
1 M
+
2
if
Nsin
x sin x dx

Q.38

( ,2)

4 2

cos2x)
x (sin2x
Q.40 dx
(1+sin 2x)cos2
0
x

Q.41
that

Prove

du

f (t) dt

du.

f(u).(x

u)

k|

dx
+ 4cosx)2

(5

Q.42

Q.43 Evaluate
x dx

1n

( ln

Q.45 Lim n2 (2006sin


+x
that nf( 1 n

2007cos x)| x |dx .

)
1 x+

16

1+ Q.44

1
1

tan x 1dx

a x lnx
a x dx
Q.46
+ ).
dx = lna.
Show
+ ).
0

x a

x f(

x a x

(2x332 + x998 + 4x1668 sin x691)


Q.47 Evaluate the definite dx
1+ x666
1
integral,
Q.48 Prove

that
(2)

(a) (x
(b) x dx =
x

)(
x) dx = 8
(
(c)

dx
=
(x)(x)

Q.49
=
Q.50
x)dx .

Q.1
Q.2
Q.3

Q.4
Q.5

4 cos x

(cosx+ 1)2
1)2
(cosx 1)2
1
1

Evaluate
1):2 exln tan
If

f(x)

where , >

(a)
(b)

x.dx

=(+)
(x)(x)
2

1
1
2
,
find
+

f(x) dx
(cosx
(cosx
2

+ 1)2 cos
2
x
(cosx
sin (cos

where

<

EXERCISE3
1

sinx dt

Find the range of the function,


1 2t cosx +.
f(x) =
1
t2 , 1] as f(x) = 21x sin 1 cos ; x
A function f is defined in [1

x
x
0 ; f(0) = 0;
f (1/) = 0. Discuss the continuity and derivability of
x
fLet
at f(x)
x ==0.
1 if 2 x
and g(x) =
f(t) dt. Define g (x) as a function of x and
x1 0 if 0 < x test the

[2

continuity and differentiability of g(x) in (2, 2).


Prove the
inequalities:
1
2
< 2
dx

(b) 2 e1/4 <


(a)
<

8
ex2x
4
6
x2 x 3
0
0

dx <
2e.

dx

< b then find a &


(c) a <
10+3cos b.
x
0
Q.7

If the derivative of f(x) wrt xthen show that f(x) is a periodic


is cosx
f (x) function .

Q.6

(d)

dx
(d) 1 2+ x2 5
6
0
2

Determine a positive integer n 5,


such that
dx = 16 6e.
0

ex (x 1)n

4x3
8x7
1the 2x
Using
If xcalculus
< 1 then find the sum of1+
x + 2+
+
+...... .
1+ x +1 4 x + 81
series
1 2x 2x 4x3 4xx3 8x7
1+ 2x
If x < 1 prove
+
+...... =
.
+
1+ x + x2
1 x + x2 1 x2 + x4 1 x4 +
that
x8

x
x
1 x
1
1
1
=
Prove the identity f (x)= tanx + 2 tan22 + .... + tan2n1 2
n1
2n1cot
2 2 2
tan
+
x
x (x t) (t) dt. Then find the value
of
Q.8 If (x) = cos
0
(x) + (x).
x
2
1
d y+ a2y = f
Q.9 If y = f (t)sin a(x t)dt then prove
a that
0
(x).
x
dx2
lntdt
dy
Q.10 If y =1 x , finddx at x = e.
(c)

x
2cot 2x
2n1

Q.11 If f(x) = x+ [xy + xy] f(y) dy where x and y are independent


variable. 0Find f(x).
dy
Q.12 A curve
C is defined by:
= ex cos x for x [0, 2] and passes through the
1
dx

origin.roots
Proveofthat
the the
function (other than zero) occursin the ranges
< x < and
2
2
3 < x < 2.
x
f(x)

Q.13(a) Let g(x) = xc . e2x & let


f(x) = e2t . (3 t2 + 1)1/2 dt . For a certain value of
g(x)
0
'c', the limit of
as x is finite and non zero. Determine the value of 'c'
x
and the limit.
t2 dt

a+ t

0
(b) Find the constants 'a' (a > 0) and 'b' such
= 1.
x0 bx x
that, Lim

Q.14
Lim

d
Evaluate:
x+ dx

3 x

1
2sin
x

3t4 +
1
dt
(t 3)(t2 +3)

sin

d2 U

Q.15 Given that


U = {x(1 x)}n & n 2dx 2 = n (n
1) Un2 2n(2n
n
prove that
1)Un1,
1
further ifnV = enx . U dx, prove that
when
n
2, V + 2n(2n
1).Vn1 n(n
n
1) Vn2 = 00
nt
n 2
dt =
Q.16 If
(x > 0) then show that there can be two integral values of x
0 x2 +t 2
satisfying this
4
equation
.
1x if 0 x
x
Q.17 Let f(x) =
1 1 < x 2 . Define the function
F(x) = f(t) dt and show
0
if

0
that
F
is

x3
(2 x)2 if 2 <
continuous in [0, 3] and differentiable
Q.18 inLet
f3).
be an injective function such that f(x) f(y) + 2 = f(x) + f(y) + f(xy) for all non
negative(0,
real
x&
y with f(0) = 0 & f(1) = 2 f(0) . Find f(x) & show that, 3
f(x) dx x (f(x) + 2)

is a constant.
2
2
2 1/n

1+
1+ 3

Q.19 Evaluate: (a)


.....

1 n 2
n
n 21+ nn 2 n 2
Lim

2
+

1 1
3n
2

+
+ .....
;
(b) Lim
n n
n
1
n
+
2 + 4 n
+
1
1n

n!1/ n
(c) Lim
;
n

nn

3nCn
= a where a and b are relatively prime, find the
Lim
(d) Given

2n
n
Cn value of (a + b).
b

, k
N and
Q.20 Prove that sin x + sin 3x + sin 5x + .... + sin
sin kx
(2k
sin x hence
1) x =
/2
2

1 1 +1 +
1
k x dx = 1+
prove that , sin
.
sin x
2k 1
3 5 7
0
+......+
/2

sin 2nx
Q.21 If nU = sin 2 dx , then show that
U
,
U
, Un , ..... , U constitute
1
2
3
x
0
an AP .
Hence or otherwise find the value of Un.
Q.22 Solve the equation for y as a function of x,
x
x
satisfying
x y(t)dt = (x+1) ty(t)dt , where x > 0, given
y (1)
0 = 1.
0
1

Q.23 Prove
that :

(a)

m!n!

Im , n = xm . (1 x)n
m , n N.
(m + n +1)!
0
dx =
1

(b)

n!

Im , n = xm . (ln x)n dx = (m 1) m , n
(1)n 0
N.
+
n+
Q.24 Find a positive real valued continuously differentiable
functions f on the real line
1
such that for all x

2
) +( f2 '(t)) )dt
f 2(x) = (( f (t)
0+

e2
x
[x]
Q.25 Let f(x) be a continuously differentiable function
then prove that, [t] f(t) dt =
k=1
[x]. f(x)
[f. ](k)
where
denotes the greatest integer function 1
and x > 1.
Q.26 Let f be a function such that f(u) f(v) u v for all real u & v in an
(i)
Prove
f is continuous at each point
interval
[a, that
b] . Then:
of [a, b] .
b
(ba)2

(ii)
Assume that f is integrable on [a, b]. f(x) dx (ba)
2
Prove that,
a f(c)
, where a c
1
x
b
Q.27 Let F (x) 4 + t2 dt and G (x) 4 t dt then compute the value of (FG)' (0)
where dash
=
x
1
+
denotes the =
derivative.
2
Q.28 Show that for a continuously thrice differentiable
function f(x)
1x
f(0).x2
2
f(x) f(0) = xf (0)
+ f (t)(x t) dt
2
2
0

+ n
m
1
1
Q.29 Prove that
= ( 1)k m
( 1)k( n)
k( ) k + n
k= 0

k + m + 1k =

1 differentiable for all real numbers x and that have


Q.30 Let f and g be function that+are
the following
properties
:(i)
f ' (x) = f (x) g (ii)
g ' (x) = g (x) f
(iii) (x)
(iv) (x)
f (0) = 5
g (0) = 1
(a)
Prove that f (x) + g (x) = 6 for(b)
Find f (x) and g
all x.
(x).

EXERCISE4

Q.1

Find Limit
S , if :
n
=
n

1
Sn +
2n

2
4n 1

+ ..........
2
+
4n 4

3n + 2n1
2

[REE 97, 6]

Q.2

(a)
(b)

Limit
n

(A)
(c)

cos tdt, then g (x + )


If g (x) =
0
equals :
(A) g(x) + g() (B) g(x) g()
2n

r
1

n r=1 n2 +
r
e37

The value of

(e)

(B) +
1 5

(a)

Let

4 2 esinx
d F(x) = esinx
dx = F(k)
possible
x , x > 0 .1If x
dx

values of k is
______.

f(t)dt=x
0

Prove that

Q.4
Q.5

2x (1 + sinx)
dx.
2

[JEE 97, 2 + 2 + 2 + 2
+ 5]

1 + cos x

(C) 1

1/ 2

(D)

[JEE98, 2 + 8]
[REE 98, 6 ]

dx

(2 4x)

(5 al2real number )y, [y] is the greatest integer less than or equal to y, then
If for
the value of the
3/2 [2 sinx] dx
integral

/2 is :

(A)
(C)
(B) 0
(D)
2

(b)

F(1) then one of the

+ t f(t) dt , then the value of

tan1( +
0
2)
1 dx
1
Evaluate 1 x x2
+ x2x)(1+e
(a)

1 dx 21 tan1 x dx . Hence or otherwise, evaluate the

=
1x+x
2integral
0

tan1

Determine the value


of
If

dx is _______ .

is
(A) 1/ 2f (1)(B)
(b)

(D)

sin ( ln x)

Q.3

(C) +
1 2

equals :

(d)

(D) [g(x)/g()]

+5

(C) g(x) g()

3/4

dx

is equal
1 + cosxto
:

/4

(B) 2

(A) 2
(c)

(C)

x3 + 3x +
dx
Integrate :
(x2+1)2 (x 2+ 1)

1
2

(D) 1
2

(d)

ecos x
dx
Integrate:
0 ecos x+ ecos x

[JEE '99, 2 + 2 + 7 + 3 (out of


200)]

/6

Q.6
Q.7

Evaluate the
integral

3cos2x
dx.
1 cosx
e2

(a)

log x

The value of the


1 xe
integral
e
(B) 5/2
(A) 3/2

[ REE '99, 6]
d x is :
(C) 3

(D) 5

(b)

(c)

1 1]
Let g(x) =
f(t)
1 for t
(0,
1
f(t) dt , where f is such that
2
2
0
and 0 f(t)

for t (1, 2]. Then g(2) satisfies the


inequality
(C) 3 < g (2) (D) 2 < g(2) <
(A) 3 :g (2)1 (B) 0 g(2)
2
2
2
2 <2
5
4
<
cosx
3
. sin x for |x|2
If f(x) =
. Then f(x)dx :

{
2

otherwise

(A) 0
(d)

Q.8

(a)

2+

Given

Evaluate

2n

(d)

sin n t
2

4
2

2x + 2
sin

dx.
4x2 +8x +13
9

(b)

[JEE 2000, 1 + 1 + 1 + 5]
Sn .
dt in terms of

4+2 t
.
[ REE 2000, Mains, 3 + 3 out of

100]

xdx
Evaluate1+
cossin
x
0
[ REE 2001, 3 +
5]
2

1
1 the equation x f (x)
.
Then
the
real
roots
of
(D) 0 and 1
(A) +1
(B) +
(C) +
2
= 0 are
2
Let T > 0 be a fixed real number. Suppose f is a continuous function such that
3+3T
T
for all x R
f (x + T) = f (x). If I = f(x) dx then the
3 f(2x) dx is
value of
0
3
(A)
I
(B) 2 I
(C) 3 I
(D) 6 I
2
1
2

(c)

n2

. FindLimit

Q.10 (a) Evaluate


3cos x3 dx.
0 cos x
sin
x
x
+
2
Q.11 (a)
Let f(x) = 2 t dt

n+

sin t d t = , find the


1 + value
t
of

/2

(b)

(D) 3

n t

f(e) + 1
f(1/e) = 1/2
.
1
1
+
+ ........ +
Sn = 1

Q.9

(C) 2

For x > 0, let f (x)


1 + tdt. Find the function f (x) + f (1/x) and show
1
=
that,

(b)

(B) 1

The integral
+


1
2 [x]
1
(A)
2

1+ x
dx

equals
ln1 x

(B) 0

(C) 1

(D) 2ln 1

[JEE 2002(Scr.),
3+3+3]

For any natural number m,


evaluate
x3m + x2m + xm
3xm + 6 dx , where x
[JEE 2002
>0
(Mains),4]
m

2
4
+
1
2m
Q.12 If f is an even function2x
then
f (cos2x)
cosx 2 f (sin2x) cosx
prove that
0
0 dx
dx =
[JEE 2003,(Mains) 2 out of
60]
1
1 x
dx =
Q.13 (a)
1+ x

zc
0

hc

(A)

+1
2

(B)

t2

(b)

2
If x f (x)dx =
5
then
0 f
2
(A)
5

1
2

(C)

(D) 1

(C) 2
5

(D) 1

4
t5 , t > 0, =
25

(B)

5
2

[JEE 2004,
(Scr.)]

x2

(c)
(d)

Q.14

dy
cos x.cos
.d then find
= .
2 /16 1+ sin2
dx
/3
+ 4x3
dx.

Evaluate
cos|+x
/3 2
3

If y(x)
=

(a) If

2
(A) 1/3

(b)

(c)

Q.15

x3

at x

[JEE 2004 (Mains),


2]
[JEE 2004 (Mains),

= (1 sin x), 1

3
is

1 t f (t) dt

( f
then

sin x

(B) 1 3

4]
[JEE 2005 (Scr.)]
(C) 3

(D)

(x

2
+ 3x
+ 3x + 3+ (x +1)cos(xis equal
to
2
+1))dx
(A) 4
(B) 0
(C) 4

1
1
Evaluate:
|cos x|2sin + 3coscos x
sin x dx
e

cos x 0
.
x2
1
dx is equal
2x4 2x21to

+
2x4 2x21
(A) +
+C
x2
2x4 2x2 +1
C
x
Comprehension
(C)

[JEE 2005 (Scr.)]


(D) 6
[JEE 2005,
Mains,2]

2x4 2x21
(B) +
+C
x3

1
(D) 2x4 2x2 + C
+
2x2

[JEE 2006, 3]

b
b a
Q.16 Suppose we define the definite integral using the
following
(f (a) + f (b)),

2
a
formula f(x)dx =
for
more accurate result for c (a, b) F(c) = c )a
fb(a)
(
fc(b) + f (c)). When c b= a
+

( +
f (c)
2
2 +
b
,

b
a (f (a) + f (b) + 2f
2
f
(
x
)
dx
=

4
(c))
a
/ 2

(a)

sin x dx

is

equal
to

(A) (1+ 2)
8

(B)

+
(
4
2)
1

(C) 8 2

(D) 4 2

f (x)dx

If f (x) is a polynomial and if


t a
Lim a

(b)

t a (f (t)

+ f
2)
(a)

(t
a)3

=0 for all a then the degree of f


(x) can

atmost be
(A) 1
(B) 2
(C) 3
(D) 4
If f ''(x) < 0, x (a, b) and c is a point such that a < c < b, and (c, f (c) ) is the point
lying on the curve
for which
f '(c) is2fequal
f(b) F(c)
f(a) is maximum,
2(f(b)then
f(a))
(b) fto
(a )
(A)
(B)
(C)
(D) 0
b a
b a
2b a
[JEE 2006, 5 marks
each]
1

(c)

50
5050 (1
x

100

Q.17
of

dx1
Find the value

[JEE 2006, 6]

(1 x )
0

50

dx

ANSWER
EXERCISE1
101

1+ 1+3cos

+ C
Q.1 ln2

cos2

x +
1 +c
Q.2 x5
+x+ 1

3
x 1
3 +
c Q.4
x 1)
(sin 2x + cos 2x)
+ c
ln
4
x
4
x+
16
8
(

1
x

x
Q.5 2 tan
1

+

x

x+ c
Q.6

e
x2 +2x 1 +
e

1
c cos +

1 tan
x

1
ln
(sec2
)
Q.7 (c)
(sin2 )sin
ln
+c
Q.8
ln

2
2
sin 2
tan
x+2
+
c

x
cos
2

1
t
1
a tan x
2
Q.9 a2 + b
Q.10 2ln
+
+ C when t = xx2 +
2

+
c
x + tan

2t +1
2t +

1
x
+1
b
3/2

1
2
Q.11 x+ x 2
1/ 2 + c

Q.3
tan

1
ln(cosx
4
1x

x
2

1
8

+ sinx) +

+2 3

x+ x 2+2

sin x +

sin a . ln

Q.12 cos a . arc cosx

cosa Q.13 3(
cos
1
1
Q.14
ln tan x+ x sec x + tan
+c 2
4
2
2
2

Q.16 (a + x) arc a ax + c
tan
x

Q.18 xln (lnx) + c


ln x

sin

xsin

2a

+ c 1+4tan x )
2
8(tan
x)8

Q.15


x 1 x

Q.17

9x

+c

/3

+
2 1 x arccos

x +c (x 2 +1
) x 2 +1.23ln1+

Q.19

xex 1

+
+c
ln
+ x +
1 xe 1
xex

t4 2
tan1 t + C where
2
Q.20 ln(1 x4)
Q.21 6 t + t + 1 ln(1+ t )

2 x1/6 2
4t =

+c
1
4 +2 tan1 cos x ln 1 + cos
Q.23 C ln(1 + (x +
Q.22
x
2
1+ (x +1)e
cos xx2 +c
1 cos
1)e )
2
x

11

Q24. sin sec


2
x
2
+c

1 ln (4+3sin
x +3cos x )
2
Q.25

Q.26

+ c
24
(43sin x 3cos x )
Q.27 1 ln 3 +sin x cos x + arc tan
+
(sin x +
cos x ) c
2 3
1 3 sin x + cos x 2

Q.29

ln [ cot+x cot+

sin cot x

1
2

1
x +
sin x cos x ln tan

2 8
2

1
1

+c

Q.28 n(secx) n(sec2x)


+ n(sec3x)+ c
2
3

+
Q.30 ln x sin x

cos
x
x sin x
cos2x x cotxxcos
. (ln (e cosx +

Q.32
sinx
+c

cot x+2cot

+c

x
Q.31 2x 3arctan
+1+c

tan
2

1 1 tan
1
1 ln tt22 2 t +
1 2 t + c where t =
Q.33 ln(1 + t)
ln(1 +
+
2
t
+
1 cotx
2
4
2
2
t4) +
x
1
x 1
x
Q.35 c
Q.34
lntan tan2 +c
(x2 1)2
2
2
2 4

))cos2x

1+ x
ax2+

+ c Q.39 2(7x 20)


b

+c
sin1
k Q.38 ex1 x
9
7x10
x2
2
2

lnx
|u 1| + 3 tan
1 1+2u

Q.40 arcsecx
+c
Q.41 ncx
1
x
u4 + 2u+1
3 + c where u =+ 3
x2
1
1 x

5 t 1 1
8
1
1+x
2
Q.42 tan1 t n
+ c where
sin x 1) x

2
5
3

+
t =
5
t
1

1 x

+
6x
7
2sin 2x
1
1
+ c
Q.44 4 ln x +
+ 6 tan1+ x2 + C
x
Q.43 tan sinx +
(x) +
Q.36 c ecos x (x + cosec x)

Q.37

Q.45 2
arctan
3
2
Q.47 +c.

cosx

x
3(x+1)
+c

Q.46

x
x

2xx2
2n
4 x + 2 2

+
2xx
2
x
4
x

Q.48 1
2
1

+
ln x +
+2 +

+
1

x
2 12 + C
x x 2

1 ln 2 +
t 1 1 t

2 2 t 2 1 t

ln

+ where t = cos and


(cotx)
1 cosec

Q.50 1
cosec tan

2
2
2
2x

Q.49

x2
1

2x+1

sin1

= cosec

+c

EXERCISE2

5
Q.1 2
Q.2 ln2
Q.3 6 2e
Q.4 1
Q.6 ln 2
64
8
Q.5
2
6

22
Q.12

Q.13
Q.7 1 sec(1) Q.8 2 Q.9 2 2 + 4 (3 3 2 ) 2
7

3
8
6

(1 ln 4)
2
3
(a+b)
3 2
Q.16
Q.14 4 2
Q.17
l4 n (+ 2 1)
(e2 +1)
3 Q.18 5
3
2 2
Q.15

Q.19

16 2

Q.20
2 2 5

Q.22 (a+2b)
3 3

Q.28
2a(a+b)
2
Q.33 a
4
Q.42 5
2 + 27

( + 3)

Q.23

Q.36 8

Q.43
1

Q.4


,
2 2

Q.24

5
Q.29
3

32
Q.49 2
15

Q.2

2 arc
Q.211
arc tan3tan1

3
3

Q.31
12

Q.30 32
16

Q.50

Q.27
Q.32

6 3
real & distinct k

2 (1+ ln2) +

8 41
2

EXERCISE3
Q.3 cont. & der. at x = 0

g(x) is cont. in (2 , 2); g(x) is der. at x = 1 & not der. at x = 0 .

2 x

2 + x x02 for
0<x
g(x) =< 1
2
x2 x

1 for 1 x
2
2
1
Q.7 (a)
Q.8 cos x
Q.10 1
1x
+e

Q.13 (a) c = 1 and


Limitx

will be

Q.16
4

x = 2 or

Q.17 F(x)

Q.18
U =

f (x) = 1 + x2 Q.19

2
2
Q.5 (c) a = & b =
Q.6 n =
13
7 3
61
80
Q.11 f(x) = x + x + x
119 119

3
(b) a = 4 and b
2 =1

x2x

Q.30
2e2x

R
Q.37 (a) ; (b)
ln 2
Q.39
22 ln 22
ln2
Q.40 3
4
8
16
3
16
+ 4
ln
2 3 Q.45 2007
1 Q.44
Q.47
2
3
666

Note
that
(x; + 2) for

Q.22

5250

= 12
(x2)3
+21
3

if
if

13.5

x 1
0
1< x2

2 x 3
<
(a) 2 e(1/2) ( 4); (b) 1
3
e

e e1 x
Q.24 f (x) = ex + 1
y=
x3
0
f (x) = 3 + 2e2x; g (x) = 3

Q.14

if

Q.27

ln 4; (c)

Q.21
; (d) n43
n
2

EXERCISE4
Q.1 /6
1

Q.2 (a) A (b) B (c) 2 (d) 16

Q.4 2 11n
Q.5

( a

(e)

Q.3 (a) A (b)


ln2

11+1
111

3
C, (b) A ; (c)
2
+ x 2) +

1 + x) +
tan-1 x 1 ln(1
2

lnx (1 + c, (d)
1 + x2
2
1

2
2 tan1
2
3

2
Q.7 (a) B, (b) B, (c) C, (d)
ln
2
x
2(x +1) 3
n(4x2 + 8x +13) +
Q.9 (x + 1)
Q.8 (a) 2 ln 2, (b)
3
4 C
tan
1


if( 0 ,)
1 5 1

sin
Q.10 (a)8 4
3 , (b) I

( 2 ) if
=
sin(,2)

Q.6

1
2x3m + 3x2m + m
Q.11 (a) A, (b) C, (c) B,
6(m+1)
(d)
6xm m+1
Q.13 (a) B, (b) A, (c) 2, (d)
1
tan1

4

3
1 e

2
Q.14 (a) C, (b) C, (c) 24ecos
+
1

sin 1
5
2 2 Q.17 5051
Q.15 D
2
Q.16 (a) A, (b) A, (c) A

+ C

ELEMENTARY

DEFINITE

INTEGRAL

(SELF PRACTICE)

Evaluate the following definite


integrals.
n2
sin1 x
dx
x (1x)

Q.1

/2

Q 4.

e . cos x dx

Q 5.

2x

/4

Q 7.

sin 2x
dx
4
sin4 x
cos
x

/2

Q 8.

(x 1) (2

Q 10.

dx

x)

/2

Q 11.
2

d
)

/4

Q16.

Q 18.
0

/2

Q 21.

dx
1 + cos .
cosx

sinx dx
4

/4

Q 22.

Q.24
0

/2

Q27.

Q 30.
0

dx
(1 2x)2 1 x2

dx
1+ x2

Q 25. x x4 1
(
1

1+ x2

Q 12.

x) dx
2

dx

Q17.

(x1) x2 2x

Q 20.

Q23.

dx

(1+ x2)3/2
x
dx
3 x

a2 x2

1
2

Q 29.

sin1x

)3/2

dx

1 x2

1+ x4 dx
0

Q33.
0

dx

1 + 2cosx
dx
2
(2 + cosx)

Q 39.

cos
/2

2 x2

dx
2

1x

1 x2
dx
1+ 2x + x4

/4

Q 36.

(1+x)

sin x + cos x
dx 3
(x+1) 1+ x2

Q 35. 3 + 2sin x +
x
0

3/4

Q 31. x2

cos3x sin 3x dx Q 41.


0

Q 28. x
(tan10

2
cos d Q 38.
sin
0
/2

sin x . cos x dx
2

Q 26. x a2
dx
+ x2
0

e2e2
ee1
where a =
& b=

Q 34. x5 1 x2dx
0

dx

dx
< <
x2 + 2xcos 1+where

Q 32.

Q 40.

1sin2 xdx

sin x cosx
dx
2
cos x+ 2cosx +
2

Q 37.

cos 2x

/4

x dx
x+1 +
5x+1

(0, )Q 19.

1/2

Q 9.

1
2 dx
n x

Q14. x2 . 4 x2 dx

dx
5 + 4sin x

/2

nx

a b

/2

x cosx cos3x dx

Q 15.

0
2

+
+
(1 sin x) (2 sin
x) dx
(x1) (5x)

2
2
Q 13. sin cos(a2sin2 + bcos

Q 6.

sin x dx
1 +cos2 x

/4

cosx dx

Q.3

x dx
54x

Q.2 x ex dx
0

/2

sin + cos
d
9 + 16sin 2
x + sin
x

1 + cosxdx
0

d 1
Q 42. dx 1+ e1/x


dx
1

Q 43.

dx

ln(xx ex )

Q 44. x d (lnx)
1

Q 45. If f() = 2 &(f(x) + f (x)) sin x dx =


0 f(0)

| x | dx

3
Q.46 x
Q.47 cos2
8
0
a

/2

Q.48

secx tanx cosecx


secx + tanx
1+
2cosec x

dx

5, then find

11 + x
x cos2

dx

4
8

4
1

x
f
''(x)
dx
,
where
f (x)
Q.49

=cos(tan
0

x)

n3

Q.50

n2

f (x)dx, where f(x) = e + 2 e 2x + 3 e 3x


+ ......
ANSWER KEY

2
4
1
Q 5.
6

1
e
n
2
2
2
Q6. e n2

Q 1.

Q 2.

Q 9. 6

Q 10.

b3
Q 13. 1 3 a
3 a b2

Q 17.
Q 21.

3
3

3
4

Q 18 sin

16

1 32
a2
( 2)
Q 25. ln
Q 26.
4 17
4
1
1 n2
Q 28. 4
4 2
+

Q 31.

2 2

Q 35.
4
Q 39.
2

Q 32. 1
Q 36.
Q 40.

1
ln3
20

5
12
e2 e2

Q 43. ln 2

Q 44.

Q.47

Q.48 /3

tan14

Q 7.

1
2

Q 11.

Q 14.

Q 22.

Q3. +

Q 15.
Q 19.
Q 23.

2
5
4
Q 8. ln
3

Q 4. e

Q 12.

3
16
14
15
3
2

1
Q 27. tan
4

ln

9 + 4 2
1

2n
7
1

Q 16.
3
tan1
3 2
Q 20. 3
2

Q 24. 1 n (2 + 3)
2

1
2+
2

5
2

1
Q 29. ln2
4 2
1
Q 33. ln3
2

0 ; 1 if =
Q 30. 2sin if
0

Q 37. 4

Q 38. 1

Q 41.

Q 45. 3
3
Q.49 1
2 2

Q 34. 3 +
24

2
Q 42. 2
+
1 e

Q.46
|
Q.50

|b||a

1
2

EXERCISE5
Part : (A) Only one correct option
1.

2.

3.

[f(x)g (x) f (x)g(x)] dx is equal

to f(x)
(A) g(x)

(B) f(x) g(x) f(x) g (x)

(C) f(x) g(x) f(x) g(x)

(D) f(x) g (x) + f(x) g

sin xcosx
2
(A)
+c
tanx

dx is equal to
(B) 2 tanx + c

n |x|

8.

(B) 2

1+n x
(nx+ 2) + c

(D) 2 1+ n x (3 nx 2) + c

1
sin 2x + c
2

(B)

1
sin 2x + c
2

a+x a
dx is equal to
x
a x a+ x
a2 x2 + C

(C)

(C)

1
sin x + c
2

x2 a2 + C

(D) sin x + c
2

(D) none of these

tan(x

)tan(x + ) tan 2x dx is equal

to
(A) n

sec2x . sec(x + )
+C
sec(x )

sec2x
(B) n sec(x )sec(x + ) + C

(C) n

sec2x . sec(x + )
+C
sec(x + )

(D) none of these

sec x 1 dx is equal

x + cos2x

to
cos
1
(A) 2 n
2 2 + C
2

2 1
cos x + cosx

(C) 2 n 2
2 2 + C

9.

(D) 2 tanx c

(A) 2 a2 x2 + C (B)
7.

+c

xtan1x
1+ x2 dx = 1+ x2 f(x) + A n (x + 2 + ) + C, then
If
c
x
1
(A) f(x) = tan
1
1
x, A = 1
(C) f(x) = 2 tan
(B) f(x) = tan x, A = 1
1
x , A = 1
1
8
(D) f(x) = 2 tan x, A =
sin8 x cos
x
dx =
1
2
2
1 2sin
xcos
x
(A)

6.

tanx

dx equals :

2
1 + n x (nx 2) +
3 c
1
(C)
3 1 + n x (nx 2) +

5.

(C)

x 1 + n |x|

(A)

4.

(x)

cos

dx
x sin2x

x +
x

1 +
(B) n cos
cos2
2 2
2
C
(D) none of these

is equal to

(A)

cosx + tan5x/ + C
2
5

(B)

(C)

1
2 tanx
5
2

(D) none of these

tan5x/+ C

1
tanx +
2
5
2

tan5x/+ C

10.

3x4 1

Primitive of

(x4 +

x
(A)
+c
x4 + x 1
+

12.

x
+c
x4 + x 1+

(B)

x+ 1
x4 + x + 1 +

(C)
11.

w.r.t. x is:

x +) 1

If

c x4 + 1

x (x2 +

(D) x + 1 + c

x4 + x 1+

B
+ c, where c is the constant of integration
1 + x2 then:

dx = A n x+

(A) A = 1; B1=
1
(C) A = 1; B = 1
1 x
dx equals :

(B) A = 1; B = 1
(D) A = 1; B =
1

1 +x

1 x 2 1 x 1+( x )+ c

(A) x
cos
(C) x
13.

14.

cos
1 x 2 x

( )

( )

(B) x
(D) x

1 x + 2 1 x + cos
1 x +2 1 x 1( )
x +c

x +c
sin x. cos x. cos 2x. cos 4x. cos 8x. cos 16 x dx equals:

sin 16x
(A)
+c
1024
1

cos 32x
(B)
+c
1024

cos

x +c
cos 32x
(D)
+c
1096

cos 32x
(C)
+c
1096

d x equals :

cos6 x + 6sin
(A) xtan 1 (tan x + cot x) +
15.
16.

(B) tan 1 (tan x + cot x) +


c tan 1 (tan
(C)
x cot x) +
c
c
(D)

dx is equal to:
ln(1+ sinx) + xtan
tan 1 (tan x cot x) +

4 2
x
c

(A) x n (1 + sinx) + c (B) n (1 + sin x) + c


(C) x n (1 + sin x) + c (D) n (1 sin) + c

dx

equals:

cos x .
sin2x
2
(A)
(tan x)5/2 + 2 tanx + c
5
(C) 2 (tan2 x + 5)
5
c
17.

18.

If

dx

(B)

2tanx +

tanx +

(D) none

x + c where c is an arbitrary constant of integration then


the
tan
sin x cos
values
x of a and b are respectively:
2
2
2
(A) 2 &
(B) 2 &
(C) 2 &
(D) none
3
3
3
3

= a cot x + b

x2 1

[IIT - 2006, (3, 1)]

x3 4
2
2x 2x
+1 dx is equal

(A)
(C)

to 4 2x2 +1 + c
2x
x2

(B)

2x4 2x2 +1 +
c x3

2x4 2x2 +1 + c

(D)

2x4 2x2 +1
+c
2x2

x
Part : (B) May have more than one options correct
19.

2
(tan2 x + 5)
5
c

If

x 1 dx

( )

is equal to

x2 2x2 2x +

2
(A) 1f(x) = 2x
2x + 1

f(x)
+ c then
g(x)
(B) g(x) = x + 1

(C) g(x) = x
20.

5+

(D) f(x) = 2x2 2x

dx
= tan1m tan + C then:
4cosx
2

(C) l = 1/3

(A) l = 2/3
21.

3cot3x cotx

If tanx 3tan3x
then

3 tanx
3 + tanx

p = 1; q =

(B)

p = 1; q =
; f(x) = x; g(x) = n
3 + tanx
3

; f(x) = x; g(x) = n

3 tanx

2 ; f(x) = x; g(x) = n 3 + tanx


3 tanx

p = 1; q = 3

(D)

1 ; f(x) = x; g(x) = n 3 + tanx


3 tanx
3

sin4

sin2x
4q =
p =+1;cos
x
x dx is equal

2
(A)to:
cot 1 (cot
x) +
2
c tan 1 (tan
(C)
x) +
c

23.

(A)
24.

(D) m = 2/3

dx = p f(x) + q g(x) + c where 'c' is a constant of integration,

(A)

(C)

22.

(B) m = 1/3

2
(B) cot 1 (tan
x) +
tan
1 (cos 2x) +

(D)
c
c

x 1

dx equal:

x 1
x+ 1
x+ 1
1 2x 1
1
1
1
n
(B) n 2
(C) n 2 x 1 + c (D) n 2
x+1 +c
x+1 +c
x1 +c
2
4
2
4
n (tanx)
dx equal:
cosx

sinx

1 n2
(sec x) + c
2
1
(D) n (cos x cosec x) + c
2 2

1 2
n (cot x) + c
2
1
(C) n 2 (sin x sec x) + c
2
(A)

(B)

EXERCISE6

1.
2.
3.

4.
6.

Integrate with
respect

cosec2 x.sinx

(sinxcosx)

. dx

1 x2
Integrate with respect to 1 x2 + x4

x
1
Integrate with respect to
2
x
(x +1) x +
2
(x 1)2
dx

x4 + x +
3
tan + tan

1
d1 + tan3

2 sin 2 cos
d
6 cos2 4 sin

5.

7.

+ cos4x
cos5x
dx
12cos3x

8.
10.

3 + 4sin x + 2cosx

dx

3 + 2sin x + cosx
dx
(x (x ) (x

14.

x3 x +
x
dx
e2
2
(x2 + 1)

{ ( )
}dx

x 42+
x2+1 ln

x
1 2lnx

16.

18.

20.

22.

23.

11.

13.

)
12.

9.

cos ec x cot x sec x


dx
cos ec x + cot x1 + 2sec x
.
2 x x2
dx
x2

a + bsinx
dx
+ asinx
2 )

b
1 + xcosx

2sinx

1 + cos cos x
dx
cos + cos
x
dx
(x3
+ 3x2 + 3x1) x2 + 2x
+
3
(cos2x 3)
4

cos x 4 cot
x
15.

a (+ dbx

, (a > b)

cosx)2
17.

19.

21.

(7x
10
tan
dx.

3/2

2
x )

dx

2 x )
x. n (1 +

dx
+1

x4 x3

dx

dx

1 x2
xcos +
f(x)

dx =
+ c then find f(x) and g(x)
1
2
g(x)
(x + 2xcos )+ 1
3/2

n (1 +sin2 x)

24.

Evaluate

25.

Integrate,

26.

cos x

dx.

x3 + 3x 2
+
dx.
2
(x2+1) (x +

[IIT - 1999, 7]

1)

For any natural number m, evaluate,

(x3m
+

+ x2m
xm

) (2x2m

> 0.

+ 3xm + 6)1/m d x, x

[IIT - 2002, 5]

ANSWER
EXERCISE5
1. C

2. A

3. A

4. A

7. B

8. C

9. B

10. B

13. B

14. C

15. A

5. B

x
c

12. A

17. A

14.

9x3

18. D
15.

24. ACD

1
3

2a
bsinx
) + bcosx) (+a2 2b
a2 b (a

2 2
1

+t
+
t
1

3 3 9

18.
+c

19. x tan
x

+
x2 1
1 tan 1
2x2 1
2

tan
1

x
3

3
+c

tan +
1+

2 tan
+c

1 3

1
3

9.

10.
11.

2x tan
1
1 +2
x2 +

cosx
b + asinx +
c
20. 2

1
x3+1
1
n
3
21.
x3
) +c
3
3x
3(x3 +1

tan 1

8.

1
x. n (1 +2) + (tan x)
2
+ n (1 + x
) n
c

6. 1 n1+tan+ 1
6
3 ntan

4 x + 2 2 2 x

2 x x2 2

+
ln

x
2
x
4
x

2x+1
+ c
sin1
3

3
1
(sin2) + c
5. 2 n sin2 4sin + 5 + 7 tan

7.

)3 / 2

1
x
16. sin1
sec2 + c

2
2
1
2(7x
x+ 3
17.
+ c
1
x
9 20)
7x 10
+c
+ c 2. 2 3 n
1
x+ + x
x2
x

1
where t=
x +1
4.

3ln
. 2
1 +

x
2

a b
x
arctan a + b .tan + c
2

EXERCISE6

3.

(x2 +1) x2 +1

19. AC 20. AB 21. AD 22. ABCD 23. BD

x
1. n 1+ 2tan
2

1
2
13. c 3 tanx.(2 + 2x).
4 cot x
tan

6. A

11. C

16. B

x + 1

12. ex
2+

22. ln(x e
+c
sinx

)-

ln (1 - x e
2

2 sinx

sin2x
(sinx+
)+c
2

23. x; x + 2x cos + 1

x
2x 3arctantan
2
+1 + c

24. tan x ln (1 + sin x) 2x 2 tan ( .tan x) + c.


2
+
1

1 ( x)
cos
2

+ c
1 ( + x)

cos
2

x cos + sin n
2 x +
. c x

x2 + 2x 3
2
( + 1)
8 x

1 . cos1 2

+16
x + + c
1

25.

x
3
1 2
1
tan x - n(1 + x) + n (1+x 2
+c
)
+
1
+
x2
2
2
4
-1
m+ 1
z m

26. 6 (m + 1) + c, where z = 2x
+ 3x
3m

+ 6x
2m

EXERCISE7
Part : (A) Only one correct option
1.

1 2
If f(x) is a function satisfying

x+ x f(x) = 0 for all non-zero x,
f
then
2
(A) sin + cosec
(C) cosec
(B) sin
1

2.

The value of the


integral
(A) sin

f(x)

sin

dx equals

(D) none of these

dx
, where 0 < < , is equal
2 + 2xcos 1
+to
2

(B) sin

2sin

(C)

(D) 2 sin

( +)
If
=

f(x) dx = a,
r = 1 f r 1 x dx

0
(A)then
100 a
0 (B) a
(C) 0
100

100

3.

cosec

(D) 10 a
x

4.

T T
2, 2 and has period T, then f(t) dt is

If f(x) is an odd function defined

a
on
(x) =
T
(A) a periodic function with
(B) a periodic function with period
2
period
T
T
(D) a periodic function with
4
(C) not a periodic function
period

5.

If f() = 2 and(f(x) + f(x)) sin x dx = 5 then f(0) is equal to, (it is given that f(x) is continuous in [0,
0 ])
(A) 7
(B) 3
(C) 5
(D) 1

6.

If f(0) = 1, f(2) = 3, f(2) = 5 and f(0) is finite, then


to
0
(C) 2
(A) zero
(B) 1

7.

9.

lim
sin . sin
. sin 3.......sin(n
1)

2
2n
2n
2n
n
n

(B) e

is equal to
(D) none of these

(C) e2/

4/

(A) n 2

3

(B) n 2

cosx
If A = (x + 2)2dx,
0

1
1
2 A
2 + +

/2

/3

f(x)

dx is equal to

(C) n ( 2 )

sin2x
dx is equal to
+
0
x 1
1
(B) 1
(C) 1 +
+2A
+2 A
then

(D) n ( 3 )

(D) A

1
1
+ 2
2

| x | dx
2 2x 1 has the value
+

8cos

/ 2

(A)
11.

0, .

f(x) = Minimum {tanx, cot x} Then


x
2

(A)
10.

(D) none of these

1/n

(A)
8.

x . f (2x) dx is equal

2
6

(B) 2
12
3n

Lt

r = 2n +

n
r12 n2 is equal to

(C) 2
24

(D) none of these

2
3

(A) log

(B) log

12.

If

cost

x2

sint

dt =

(A) 2sin2 x
xcos y

14.

x dx

(1 x)
15
16

16
(B) 5

Let 1 =

(A) 1 > 2

21.

1+ x2

f(x)
(D)

(D) none

16

(B) 2 >1

(C) 1 =2

(D) 1 > 2

1
(C) 2 [x]

(D) none of these

[x]

3
x2 +1
tan1 x + tan1
The value of the integeral
dx is equal to

x2 +
x
1

1
(D) none of these

(A)
(C) 4
(B) 2

/2

The value of log | tanx + cot x |


dx
is
0

If

(B) log 2

(C)

log 2

(D)

log 2
2

ex

0
) dx = 0, (B)
then
(A) 1 <(x < 2
<0

(C) 0 < < 1

n+1

f(x)dx = n
Suppose for every integer n,
n is :
(B) 14
(A) 16
1

. The value of
2

(C) 19

et
1 dt has the
a1 t a

(D) = 0

f(x)dx
(D) 21

Let A = 1e+ td dt then


0

:
a
(B) Aevalue

(A) Aea t
5

22.

(D) none of these

dx
and = x , then

The value of (x [x]) dx


0
is
1
(A) [x]
(B) 2[x]
2

20.

(C)

dx

(A) log 2
19.

y2

x
1 2sin 2

18.

3
2

3/ 4

17.

2sinx2

(A)

16.

(C)

n
0 , where x n +1, n 1,
= 2, 3.....
, then the value of
If f(x) =
=
dx
1
, else
0
where
(A) 1
(B) 0
(C) 2

15.

(D) log

dy
dt , then the value of is
dx
2sinx2
(B) xcosy2

13.

2
(C) log 3

3
2

5/2

(25 x )
3

(A) 3

x4
2

(C) ae

(D) Ae
a

dx equals to :
2
(B) 3

(C) 6

(D) none

23.

dt
The function f(x) =t satisfies

[IIT - 1996]

24.

25.

x
(A) f(x + y) = f(x) + (B) f = f(x) + f(y)(C) f(xy) = f(x) +

y
f(y)
f(y)

2
cos x
,a>0
The value of 1+a dx
x
is

(A)
(B) a
(C) /2
1/2

The integral [x]

1/2

(A)

1/2
1

26.

27.

28.

(D) none of these

(D) 2

1 + x
n 1 dx equals:

(B) 0
(C) 1
x

(D) 2 ln (1/2)

n then the expression of (m, n) in terms of (m + 1, n 1)


If (m, n) = tm(1+ t)dt,
is
0

[IIT - 2003]
2n
n
n
(A) m +1 m 1 (m + 1, n
(B) m +1 (m + 1, n
+
1)
1)
2n
n
n
(C) m 1 m 1 (m + 1, n
(D) m +1 (m +1, n
+
1)
1)
1
1
f
If t2 (f(t)) dt = (1 sinx), then
[IIT - 2005]
is
3

sin x

(A) 1/3
(B) 1/ 3
(C) 3
(D) 3

{x3 + 3x2 + 3x + 3

2equal to
(A) 4

(x +1)cos(x
+

[IIT - 2005]

1)} dx is

(B) 0

(C) 4

(D) 6

Part : (B) May have more than one options correct

29.

The value of integralxf (sinx) dx is

(A)

/2

(B)

f (sinx) dx

30.

dx

(C) 0

(D) none of these

f (sinx)
2

f(x) dx is equal to
If f(x) is integrable over [1, 2], then
1

lim 1
(A) n
n
(C) lim 1
n n

r=
n

r=

31.

If f(x) =
to
0

lim 1
(B) n n

+
fr

n
1

1
(D) nlim
n

(cos t + sin t) dt, f(x +

(A) f(x) + f()


32.

r
f
1 n

(B) f(x) + 2 f()

2n

r=n+

r
f
1n

2n

r

f
n

r= 1

) will be equal

(C) f(x) + f

2x2 +3x+3
+ 2) dx is:
0 (x+1 )( x 2
2
x+

1
cot1 3
1
1

(C) 2 ln2
(A) + 2 ln2tan
(B) + 2 ln2tan
2
3
4
4

(D) f(x) + 2f

The value of

(D)

+ ln4+cot1
4

33.

Given f is an odd function defined everywhere, periodic with period 2 and integrable on every interval.
Let x

g(x) = f(t) dt. Then:


0

(A) g(2n) = 0 for every integer n


(C) g(x) and f(x) have the same
period
/2

34.

If =

dx

, then
1+ sin3 x

(A) 0 < < 1


35.

(B) >

dx

If In =
0

1n
(C) I =
2
8 4

1) In

EXERCISE8

If f, g, h be continuous functions on [0, a] such that f (a x) = f (x), g (a


g (x)
a
and 3 h (x) 4 h (a x) = 5, then provef (x) g (x) h (x) =
0.
0
that,
Assuming log sin x dx =
that,
0

log sin

Show that
0

7.

8.

Limit
Prove that
n

10.

log(

1
n

2p +
cos 2n
cos2p

f(u).(x

u) 6.

Prove that
0

9.

+ xn

0 (

dx

xn)1/n

(n > 1)

x t. cos t dt where x is any real

Evaluate
0
number

cos1 2x
+
1 3

x2 +1tan
1

dx

p
2
3

p + r
+ cos2p + ...... +

2n
cos 2 =r = 1 4 r
2n
2p

2 x
a2 cos2 x b2 sin

du =
dt

x dx

1 3

11.

2 sin d .

a x lnx
a x dx
f( + ).
dx = lna . f( + ).
x a x
x a x
0

Prove that f (t)


0 du.
0

x) =

log 2, show

3
=

5.

+ 1
2
8 4

(D) I = 5
3
16 48
(B) I =

ecos
x cos (2n + 1) x dx, n

4.

(D) > 2
2

3.

(C) <

2.

; n N, then which of the following statements hold


good?

(A) 2n In + 1 = 2n
1 + x2 + (2n

1.

(B) g(x) is an even


function
(D) none

2x
1
dx
ex + 1 x2

2 sin (pt) sin (qt) dt,


Evaluate, I =
0 if:
(i)
p & q are different roots of the equation, tan x =
x.

(ii)

p & q are equal and either is root of the equation tan x =


x.
x
sinx
x +1 dx 0 for x
Prove that
0 0.

12.

13.

f(x)dx
Let f(x) be a continuous functions x R, except at x = 0 such
that, a R exists. If
0

g(x) =

f(t)
g(x) dx =
t dt, prove that dx
0
0

sinx x (0, ], prove that,


If f(x) =
x
2

14.
15.

Let

1 3

x4
1x4

/2


dx = f(x) dx
f(x)
x
2
0

cos1 2x

1+x2
dx.

17.

f(x)

f
4 2 esinx
d F(x) = esinx
2
, x > 0. If

dx = F(k) F(1) then one of the possible values of k is ______.


x
dx
1
x

1 3

16.

Evaluate

|cos x|

[IIT - 1995, 5 + 2 +
2]

1cosx
2sin cosx + 3cos

2
dx.2

sinx

[IIT - 2005, 2]

(1 x

50)100

The value of 5050

18.

is

dx

x
(1
dx

[IIT - 2006, (6, 0)]

50)101

ANSWER

EXERCISE7
1. D

2. C

3. B

4. B

5. B

EXERCISE8

6. C
1. 0

7.

13. C

8. D

14. D

19. C 20. C

9. A

10. B

15. B

21. B

11. B

16. A

22. A

12. B

17. B

23. C

9.

26. A

27. C

28. C

10.

35. AB

11. (i) 0 (ii)

2 3

29. AB 30. BC
17.

31. AD 32. AD 33. ABC 34. BC

4a3 b3

2
cos x for 0 < x < 1 ;
2

15. 16
25. A

2 a2+b2

2 for x 1 & 2 for x 0

2
2

18. A

24. C

8.

24
5

16.

(
n 2+

p2
1 + p2

3) +
2

12

1 1

1
ecos +esin
22
2

3
18. 5051

ASSERTION AND REASON


Some questions (AssertionReason type) are given below. Each question contains Statement 1 (Assertion) and
Statement 2
(A)Each
Statement
1 is 4
True,
Statement
2
is and
True;(D)
Statement
2 is
a correct
Statement
1. choice :
(Reason).
question has
choices
(A), (B),
(C)
out of which
ONLY
ONE isexplanation
correct. Sofor
select
the correct
1 is True, Statement 2 is True; Statement 2 is NOT a correct explanation for
Choices(B)
are Statement
:
Statement 1.
(C) Statement 1 is True, Statement 2 is False.
(D) Statement 1 is False, Statement 2 is True.

INDEFINITE & DEFINITE INGEGRATION


129.

130.

131.

Let F(x) be an indefinite integral of


Statement-1:
The function F(x) satisfies F(x +) = F(x)
cos x.
real x
Statement-2:
cos (x +) = cos
2
x.
1
2
Statement-1: |x|
dx can not be found while | x |dx can be
found.
1
Statement-2: |x| is not differentiable at x
= 0.
1
1
2
dx
dx = tan1 (x ) + Statement-2:
Statement-1:

1+ x

1+ x

132.

dx

= tan x +
C
2

log(x
Statement-1: If y is a function of x such that y(x y) = x =
x3y 2
then
y)
2

dx

x3y =c log (x 3y) +

Statement-2:

x2

133.

Statement1 : f(x) = logsecx

134.

Statement1 :

x9/ 2

Statement2 : f(x) is
periodic

2 ln x11/ 2
+ 1+ x11 +c

1+ x dx=11
11

dx

1+ x =ln|
+c

Statement2 :

x+ 1+ x |

135.
G.I.F.

10

Statement1:

tan
1 x dx=10 tan1 ; where [x] =

Statement2 : [tan x] = 0 for 0 < x < tan 1 and [tan


/ 2
x < 10.

dx
136. Statement
=
3
4
1 :
0 1+ tan x
a

Statement2 :
x)dx
0
/ 2

f(x) dx=

f(a+

/ 2

dx
=
=
3
3
4
1+ tan x 0 1+ cot x
dx

dx= f (a x) dx
f(x)
0.
0

137. Statement1 : 1sin2

0 0

x] = 1 for tan 1

xdx=

Statement2 :

x dx=
cos
.
0

138.
x+ c

Statement1 :( ex tan2x+sec
)

x
Statement2 :

dx= e tan

e (f(x)+ f(x))dx= e f (x)+

c .
139.

xx

Statement1 : If f(x) satisfies the conditions of Rolle's theorem in [,], then

dx=

f(x)

Statement2 : If f(x) satisfies the conditions of Rolle's theorem in [,], then

dx=0

140.

141.

f(x)

Statement1 : [| sin x
0
equals
8.

|+| cos x |]dx , where [] denotes G.I.F.

Statement2 : If f(x) = |sinx| + |cosx|, then 1 f(x)


2 .
n+1
=
Let f(x) be a continuous function such
3
that
n

Statement1 :

dx=27

f(x) dx
nI

f(x)

n ,

Statement2 :

dx=27

f(x)

142.

(nx)n dx, n

Let I =
n

StatementI :1 I 2. I ,3 I . . . is an increasing
sequence.
StatementII : n x is an increasing function.
x

143.

Let f be a periodic function of period 2. Let g(x) =


g(x).
0

f (t) dt and h(x) = g(x + 2)

Statement1 : h is a periodic
Statement2 : g(x + 2) g(x) =
function.
g(2).
ex
144. Statement (1+ x log x)dx= ex log
x x+ c
1 :
Statement2 : x e (f(x)
f (x))dx= +exf(x)
c .
1 +
1/ x
dt
dt
145. Statement1 : IfI
and I
2
, x> 0 then I = I
1+ t2
1+ t.2
=
x
1
2

Statement2 :
[x]}dx= 02

min.{x[x],
6

146. Statement1 : 8 < 2x


dx<12.
4
Statement2 : If m is the smallest and M is the greatest vlaue of a function f(x) in an interval
(a, b),
b
b
then the vlaue of the integral
f(x)dx is such that for a < b, we have

M(b a) f(x) dx
M(ba) .
a
a
eax
147. Statement1 : eax sin bxdx= (asinbx bcosbx) Then A is a2
A +c
+ b2
1+
sin
x
cos

x
Statement2
: ex
dx = e tanx +
2
x

cos x
c

148.

149.

d(x 2+1)

Statement1 : x2

2
+ 2+c

+ 2
2
x
a/2
Statement2 :
is 2/11 ln |x ++ 1 x11 |
1+ x11dx
+c
/3
b
b
1
Statement1 :
is
/12
Statement2
:
f
(x)
dx=
1+ tan x

f(a+ b x)
dx
a
a
/6

150.

is equal to

f (x) dx

Statement1 : If f satisfies f(x + y) = f(x) + f(y) x , yR then


=0
5
a

Statement2 : If f is an odd function then


=0
a

f (x) dx

151.
152.

153.

Statement1 : If f(x) is an odd function of x then f (t) dt is an even function


of (n)
a
Statement2 : If graph of y = f(x) is symmetric about yaxis then f(x) is always an even
function.
Statement1 : Area bounded by y = {x}, {x} is fractional part of x = 0, x = 2 and xaxis
is 1.
1
1
1 0, x = 2 is 2 sq. unit.
Statement2
x=
+ by y = |sinx|,
+....+
Statement-1: : Area bounded

=
1 4n2 22
3n 3
4n n 1 fr 1
Statement-2: n =
f(x) dx , symbols have their usual

lim
r=1
meaning.
n
n n
5
Statement-1: If
n I = tan x0dx, then
4
65 (I + I ) =
tan x .
tann1x
4
lim

154.

155.

- In-2 = I , nN.
n
Statement-2: If
I = tan x dx, n
n
then
dx
will be of the type tanStatement-1: If a > 0 and b2 4ac < 0, then the value of the
integral
ax2+ bx+ c
1

x+ A

B + c

where A, B, C, are constants.


2

156.

Statement-2: If a > 0, b 4ac < 0 then ax + bx + c can be written as sum of two


squares.
ex
x2 x+1
x
ex dx=
Statements-1:
Statements-2: e
+c
= e f(x) +
(x2 +1) 3/ 2
x +1
c
x

157.

Statements-1:

158.

(f (x)+f(x) dx
x2 2 2
dx = log |tan-1 (x + 2/x)|
+c
x2 +
2
(x 4+5x2 + 4)tan
1

x
dx
1
x
Statements-2:
a2 + x2=a tan1a +c
x
ln
x

Statements-1:
(ln ex)2= ln x+c
Statements-2: e (f(x) + f (x)) dx = e f(x)

+ c.

1 1+ 1 +c Statements-2:
For integrationx by parts we have to follow
x
1 dx=
ILATE rule.
x
4
2
x3 1+ x4

159.

Statements-1:

160.

Statements-1: A function F(x) is an antiderivative of a function f(x) if F(x) =


f(x)
2
2 2,
Statements-2: The functions
x + 1, x
2 are all antiderivatives of the
x +
function 2x.

161.

x
dx = b a
x <b

Statements-1:

Statements-2:If f(x) is a
c
b
c
b

= f(x)dx+
f(x)dx

(x)dx
a

function continuous
then

every

where

in

the interval

(a,

b)

except

162.

, a

x
3+
30

Statements-1:

dx 2

Statements-2: m and M 3be the least and the maximum value of a continuous
function
b
y = f(x) in [a, b] then m(ba)
f(x)dx

M(ba)

163.

1<
ex dx< e

Statements-1:

Statements-2: if f(x) g(x) h(x) in (a, b) then

h(x)dx
164.

x
1+
1.2

Statements-1:

f(x)dx
a

g(x)dx
a

dx <

4 functions f(x) and g(x), integrable on the interval


Statements-2: For any
(a,b),
then
b
b
b

f (x)g(x)dx
a

(x)dx

f (x)dx
g
2

165.

1
Statements-1:
x2 dx=2
1

Statements-2: If F(x) is antiderivative of a continuous function (a, b) then

F(a)

cos x
can be integrated by substitution it
(1+ sin x)sinx
2
= t.

166.

Statements-1:

167.

Statements-2: All integrands are integrated by the method of


substitution only.
1+ sin x

dx = ex tan x+
Statement-1 : excos 2

168.

cos x
c
x

Statement-2 : e (f(x) + f (x)dx = e f (x)

+c
ex(x 1)cos
(x.e
)dx= +
1 1
2
x
Statements-1:
+
4

x.e
2
Statements-2:f((x))'(x)dx,{(x)=
x

169.

f(x)dx = F(b)

sin 2(x.e )+
C
t} equals f

Statements-1:

(t)dt .

du

x log
x vdx
x+ dx
c

dx

x
2 +4x + 2
e
x

Statements-1: ex
x2 +4x + 4dx=(x + 2)2+ C Statements-2:ex

vdx+

log xdx=
Statements-2: uvdx=
u
170.
171.

x x2 1 x2
=2
3+ | x |
3+ | x |
0

Statements-1: sin

f (x)+ C
(f(x)+a f '(x))dx=
a
a
Statements-2:f (x)=
dx=
f
(x)dx+
e

0
0
af(x)dx

x =

172.

Statements-1: The value (1 x)(1


of
exceed
0

x )dx can not

+ +

m(ba)

3
Statements-2: If m f(x) M x [a,
b] then

(ba)M
173.

15
8
f(x)dx

(sin x)5/ 2

= 9 sq.
2
dx=
Statements-2: Area bounded by y = 3x andunits
y=
5/2+ (cos
4 x is
0
2
x)5/2
x
10x
9+10
log
f(x) dx=log| f(x) |
10
e
Statements-1:
+c
10 10x+ x10 dx = log|10 x + x | + c
Statements-2:
f (x)
ex(1+ x)
x
Statements-1:
= tan (xe ) + Statements-2:
cos2 (xexdx
sec
c)
2
x
xdx=tan x+c
ln t dt
1
Statement-1 : f(x)
(x> 0), then f(x) =

1+ t+ -t2f
=

1
x
x
1 1 (ln x)2
ln t dt
Statements-2: f(x)
=2
, then f(x) +
=
t+1 f

1
1
1
sin x x2
2x 2 x
Statement-1
:
dx=
dx .
3 | x |
0 3 | x |
1
1
sin x
sin x =0 .
Statements-2:
is an odd function. So,
Since
3 | x |
3 | xthat
|
1
/2

(sin x)

Statements-1:

174.
175.
176.

177.

178.

n+t

Statements-1 :

| sin x |dx = (2n + 1) COSt (0

b t)
c
Statements-2: f(x) dx=

= f(x)
a

179.

f(x) dx+

c
1

x2

Statements-1: The value of the integral


[0, 1]
0

na

f(x) dx and
0

e dx

f(x)
dx=n

f (x) dx if f(a + x)

belongs to

Statements-2: If m & M are the lower bound and the upper bounds of f(x) over [a, b] and f is integrable, then m
(b
b
a)
f(x) dx M(b a).

180.

b
Statements-1:
[cot x]dx = cot1, where [] denotes greatest integer
function.
Statements-2:
f (x) dx is defined only if f(x) is continuous in (a, b) [] function is discontinuous at all
a integers

181.
182.

183.

Statements-1:

(
4

1+ x+ x2 1 x x2
f(x)dx= 0 if f(x) is an odd
+ )dx = 0 Statements-2:
afunction.

Statements-1: All continuous functions are integrable


Statements-2: If a function y = f(x) is continuous on an interval [a,b] then its definite integral over [a,
b
b] exists.
Statements-1: If f(x) is continuous on [a, b], a b and if f(x) dx=0 , then f(x) = 0 at least once
in [a, b]
a

Statements-2: If f is continuous on [a, b], then at some point c in [a, b]f(x)


f(c) =
ba a
4

Statements-1: |

184.

0 (A, B)

x+ 2 |dx=50 Statements-2:f (x) dx=


f(x) dx+

4
2

dx

f(x) dx where C
a

1+ x
Statements-2: If f is an odd function
f (x)
dx= 0
log1

dx=0
2
a
x

d
n
dn 1 (1) n!
1
x
e
ax
dx=
m!
kx
m
Statement-1 If
(e ) =n k and
thenStatement-2 :
=xn+1
n

a
m+1
dx
n
dx
n
a
eaxdx=

ekx
0
0
x
10
na
a
Statement-1 :{x[x]dx=
Statements-2:
f(x)dx=n
f
(x)
dx

0
a
0
5

185.

Statements-1:

186.

187.

Statements-1:|

2cos x | dx=

188.

Statements-2: f (x)dx
a < b.

ecosx
0 ecosx+ ecosxdx=

189.

ex[x]=
dx

x)dx
a
n

f (x)dx where a < c

= f (a+ b
a

Statements-2: ex[x ]dx=n

1000(e

0
b

1) dx

Statements-1:
1+ 2tan =x 2
0

191.

Statements-2: f(x)dx

1000

Statements-1:

Statements-1:

190.

f(x)dx+

dx

[x]

dx

Statements-2: f(x)
a

ex

dx= f (a+ b x)
a

ANSWER
129.
137.
145.
153.
161.
169.
177.
185.

1.

D
D
C
D
A
C
A
A

130.
138.
146.
154.
162.
170.
178.
186.

B
A
A
C
A
A
A
A

131.
139.
147.
155.
163.
171.
179.
187.

dx

cos(x a)cos(x =

D
A
D
A
A
A
D
C

132.
140.
148.
156.
164.
172.
180.
188.

b)
cosec (a b)log sin(x a)

(b)

sin(x b)
cosec (a b)log sin(x b)

2.

133.
141.
149.
157.
165.
173.
181.
189.

A
D
A
A
D
B
A
D

134.
142.
150.
158.
166.
174.
182.
190.

A
D
A
A
C
A
B
A

Que. from Compt. Exams


Indefinite Integral
(
)

(a) + c
(c) + c

C
D
C
C
A
A
A
A

(d)

sin(x a)

dx
=
x+ a+
x+
b 2
[(x+ a)3 / 2 (x+ b)3
(a)
c
3(b a)
2 [(x+ a)3 / 2 + (x+ b)3
(c)
3(a b+) c

cosec (a b)log cos(x a)


+ c
cos(x b)
cosec (a b)log cos(x b)
+ c
cos(x a)

[AISSE 1989]
/ 2]+

/2

(b)
(d)

2 [(x+ a)3 /2 (x+ b)3 / 2 ]


c
3(a b)
+
None of these

135.
143.
151.
159.
167.
175.
183.
191.

A
A
C
B
C
A
A
A

136.
144.
152.
160.
168.
176.
184.

C
A
C
B
A
D
A

3.

4.

5.

3 cos x + 3 sin x dx

=4 sin x + 5 cos

[EAMCET 1991]

x
27 x 3 log(4 sin x+ 5
(a)
41 cos x)
41

(b)

(c)

(d)

If

27 x 3 log(4 sin x 5
x)
41 cos41

[Roorkee 1978]
(sin 2x+ cos 2x) dx =
1
(an arbitrary
(a) c = / 4 and a= k sin(2
/ 4c and a=
x c)+ a , then (b)
the valuec=
of a and
/2
(d)
None
of these
(c) constant)
c= / 2 and a is anisarbitrary
2
constant
x 3 x 2 dx =
[AI CBSE 1985]
(1 x 2)

x+ 1
x

+ c

2x 1 2

(b)

8 x cos
x
dx =
2 sin 2 x cos 2

x
(a+

(a)
(c)
8.

dx =
bx)2

dx

1+ x

) p

22+

=
q (tan1
2 1
p 2 2 + q (tan

/2

dx =

11.

x + + c

tan
2
8
2

a dx

13.

e x
a log

b+

sinx dx =

(a) b
12.

equals

x
ce

+ c
ce x

(a) 2[sin x cos x ]+


c
(c)
2[sin x + cos x ]+
c(92 3 / 2 dx =
)
xx 2

log[q tan1 x + p 2 2+ q (tan21 x)


+ c

(d)

None of these

[IIT 1985]

sin x cos x+2


b+

(b)

2 (1+ x 3 )3 /
+ c
2
9
2 (1+ x 3)3 / 2 2 (1+ x 3)1 / 2
(c) + c

3
9
dx

(a)

sin 2x+
c

b12 x+ log(a+ bx) 1

ba 2a b a
b a+ bx

(d)

(a)

10.

(d)

sin 2x+
c

1 2a log(a+ bx)+2
1
x a
a+ bx
b
b
b2

(b)

1
b log(a+
b a+ bx

2 (p +
2
q 2 tan1 x)3
(c) 3q
c

x 1 x 2

+ c

x+ 1
2

(d) log

[IIT 1979]

2a
x+
b 2bx)+ a
2x

(c)

1 2alog(a+ bx)2
1
x+ a
a+ bx
b
b
b2

2
1

(1+

x + 1 x 2
+
+ c

x 1
2

(c) log

[IIT 1986]

(b) 1 sin 2x +

x)2
1 log[q tan1 x+
(a)
q x) ]+ c
9.

x 1 x 2
log
+
+c

x +1
2

sin

(a) sin 2x+ c


7.

None of
these

(a) log
6.

27 x+ 3 log(4 sin x + 5
41 cos x)
41

(b)

(d)

None of these

[MP PET 2002]

x + + c
tan
2 8
2

2 (1+x 3 )3 / 2 2+(1+
9
x 3 )1/ 2 +
c
3

(b)

[MP PET 1988; BIT Ranchi 1979]

(b)

a log
b+ ce
+ c (c)

x
e x

[Roorkee
1977]

e x
b
log
a
b+

(b) 2[sin x x cos x ]


+ c
(d) 2[sin x + x cos
+ c

x + + c

cot
2
8
2

(c)

x]

+ c
ce x

(d)

(d)

1
2

x
+ + c

cot
2
8

b
b+ ce
+ c
log

x
e x
a

(a)
14.

9 x

sin1x + c (b)
3
2

1 [sin1 x 2 1 x 4 ]+
+
21+ x 2
c

2 1 x
(c) sin1 x +

15.

16.

19.

21.

2 tan x
1 tan1

+ c (b)
5
5
x 2 + 1
dx =
x 4 x 2+ 1
2

(b)
1+ x+ c
(a) tan 1
x

x)
(log
=

dx

1
x+ b

(d)

1+
cot1
+ c
x 2

(c)

(b)
(d)

x dx

(b)

sin1 xx 1 x 2 + c
(c) x 1

(d)

(a)
(c)
c

x b

x 2 1
tan1
x + c

(d)

x(log x)2 2x log x


x+ c
x(log x)2 2x log x+
x+ c

(x 2 a 2

(x 2 a 2 )+ a tan
)

1
a

tan1 x / a+

x + 1
sin1 x+x 1 x 2 + c

4
2 4
2

sin1 xx 1 x 2 + c
x + 1

4
2 4

a x dx =
x

a sin1x + x a x+ c

a
a
a

x
asin1 x a 2 x 2 +
a a

sin x cos x
If x
,
sin x cos x dx
3 44
, then
=
e
1 sin 2x

x 2 1
cot1
x +

[MP PET 1991]

2x 1
sin1 x+x 1 x 2 + c
(a)

4
2 4

[UPSEAT 1999]

(x 2 a 2
)

(b)
(x 2 a ) a tan

a
1

(c) (x 2 a 2 )+ a 2 tan1[ x 2
(d)
a2 ]
[IIT 1977]
tan 2x sec 2x dx
=
1
13sec 3 2x 1 sec 2x+
1
sec c
(a)
(b) sec 3 2x+
c
6
6
2x +
2
2
1 sec 2 2x 1 sec
(c)
(d) None of
9 2x + c
these
3
1

a 2 cos
sin 2

[IIT 1971, 77]

x= sin

2 tan x
1 tan1tan x
1 tan 1

+ c (d) None of
+ c (c)

these
5
5
2 5
5

[MP PET 1991]

23.

+ c (d) None of
these
2

(b) 1 [sin1 x 1 x 2 ]+
+
2
c
2
(d) sin1 x+2 1 x 2 +

(a) x(log x) 2x log x


2x+ c
(c) x(log x)
2x log x+
2
2x+ c
(x 2 a 2)
2
The value of
dx will be
x

2 4

22.

x
9 x

(a)

20.

(c) sin1x
3

18.

4+
c
c
1
If
f(x)sin x cos x dx
log(f(x))+ , then f(x)=
2(b
2

a
2
)
=
c
1
1
(a)
(b)
(c)
a 2 sin 2 x+ b 2 x
a 2 sin 2 x b 2 cos
a 2 cos
x
cos 2dx
2
sin 2
[AISSE 1986]
=
4 sin
x + 5 cosx2

(a)
17.

+ sin1x + c
3

1 dx =

(a)

x
9 x

(b)

sin1x + x

(d)

a
a
sin1x x
a
a

a 2 x 2+ c
a 2 x 2+ c

24.

25.

26.

(b) e sin xcos cx


(a) e sin x +
c
+
(c) e sin x+cos
(d)
e cos xsin
+ c
+ c
x
x
4e +
x 6ex
dx = Ax + B log(9e 2x 4)+ C , then A, B and C are [IIT
9e x 4ex1990]
If
A= 3 , B= 36 , C = 3 log 3+
(a) constant
2
2
35
(b) A= 3 , B= 35 , C = 3 log 3+
constant
2
36
2
35, C= 3 log 3+
A=
3
,
B=
(c)
2
36constant
2
(d) None of
these
[UPSEAT 1999]
The value of sec 3 x dx will

1
[ secbe
x tan x+ log(sec x+
(a)
2 tan x)]
1
(b)
3 [ sec x tan x+ log(sec x+
1
(c)
4 tan x)]
1
(d)
8 [ sec x tan x+ log(sec x+
x1
[IIT 1983; MP PET 1990]
(x+1)
3 exx)]dx =
tan

e x
xe
(b)
+ c
+ c
(x[+sec
1)2x tan x+ log(sec(xx+
+ 1)2
e x
e x
(d)
tan +
x)]c
(c)
+ c
(x+ 1)3
(x + 1)3

(a)

27.

28.

If I=

(a) 1
(c) 5

sin 2x dx
, then for what value KI=
of e (sin 2x 2 cos 2x)+
constant
K,
x
(b) 3
(d) 7

dx

The value of
will be
3 2x x
2

(a)

1 log
3+
x 1

x3+
x
(c) 1 log

2
1

29.

x2x+ 3 dx =

[UPSEAT 1999]

3+ x
1
log

3
1 x

(d) log

1 x
x
3+

(b)

[AISSE 1985]

x (2x+ 3)3/ 1 (2x+ 3)5 / 2


2
+ c
3
15
x (2x+ 3)3+/ 1 (2x+ 3)5 / 2
(b) 2
+ c
3
15
x (2x+ 3)3 / 2 + 1 (2x+ 3)5 / 2
(c)
+
c
2
6
(d) None of these
(a)

30.

cos 2 log cos + sin

d =

2
cos
sin
(a) (cos sin) cos sin
log

cos + sin

2
(b) (cos + sin)
cos sin
log

(cos sin)2
(c)

log
cos
2

cos+sin
sin

[IIT 1994]

[MP PET 1992]

1
1 sin 2 log
+ log sec 2
2 tan
4
2
x 2
[MNR 1989; RPET 2000]
dx =
(x sin x+ cos
x)2
x sin x cos
(a) sin x+ cos
(b) x
x sin x+ cos
x sin x+ cos
xx
(c) sin x x cos
(d) xNone of
these
x
x sin x+
If u=
e ax cos
cos bx dx and v=
e ax sin bx dx , then (a
2
2
2
2
x
2
(b) (a +
2
b )e
+ b )(u + v )=
(a) 2e ax
(d)

31.

32.

33.

2ax

(d) (a
2 2b )e

(c) e 2ax

2ax

If In = (log x)
dx, then In + nIn1 =
[Karnataka CET 2003]

(a) x(log x)n


(c) (log x)n1
34.

35.

36.

37.

38.

dx
sin x
2 =4
(a)
2 cos
+ e x /
+ c
x
2
x+ c
(c) e x / 2 sin
2

(b) (x log
x)n
(d) n(log x)n
[Roorkee 1982]

x/2

(b)

2e
x

x /2

cos
+ c
2
x+ c
/ 2 sin
2

(d)

2e

2x+ 3
dx = 9 ln(x 3) 7 ln(x 2)+ A ,
5x+ then
6
A=
5
ln(x

2)+
(a)
(b) 4 ln(x 3)+
constant
(c) constant
Constant
(d) None of
these
dx
=
2+ cos
x
1
1

x
2

(a) 2 tan1
tanx +
tan + (b)
tan
1
3 c

3
c
2
1 tan1
3

1
x

2
(c)
tan + c (d) None of
3
3 these
2
x
[MP PET 2004]
equal to
x x 2 + dx
+
1
2x 2+
2x
1 tan1
1
+ 2 1
4

(a)
(b)
1

tan
1
3
3

3
1 tan1(2x 2 +
(c)
(d) None of
these
3 1)
dx
[IIT 1984]
=
(sin x+ sin
2x)
1 log(1 cos x)+ 1 log(1+ cos x) 2 log(1+
(a)
6 2 cos x)
2
3
(b) 6 log(1 cos x)+ 2 log(1+ cos x) 2 log(1+
2 cos x)
3
(c) 6 log(1 cos x)+ 1 log(1+ cos x)+ 2 log(1+
2 cos x)
2
3
(d) None of
these
5

2x+ 3

a
2 (x 2+1)
1 tan1 ,
If
dx = log

e (x1)
If

[MP PET 1992]

39.

(x1)(x
2

+1)

x+ A 2

where A is any arbitrary constant, then the value of


a is
(a) 5/4
(b)
5/3

[MP PET 1998]

40.

(c) 5/6
(d) 5/4
x+ 1a

(2x +
2 1) dx
= log
If

4)
(x
2

(x 2
2
x+
1 x
2
1)

x
(a) 1/2, 3/4
(b) 1, 3/2 + C, then the values of a and b are

(c) 1, 3/2
(d) 1/2, respectively

(
1.

If I is the greatest of the definite


integrals
1

I1 = ex cos
0 I2
1

I3 =

x dx, =
1

ex dx, I=
4
cos 2 x dx
ex e/ x
2dx,
then
0

(a) I= I1
(c) I= I3
2.

0
1

Definite Integral

[Roorkee 2000]

(b) I= I2
(d) I= I4

g(x)= x 2.
Let f(x) be a function satisfying f(x)= f(x) with f(0)= 1 and g(x) be the f(x)+
function
satisfying
The value of
1
integral f(x)g(x)dx is equal
0 to

[AIEEE 2003; DCE 2005]

1 (e 7)
(a) 4

1 (e 2)
(b) 4

1
(e 3)
2

(d) None of
these

(c)
3.

4.

If Im = (log x)mdx satisfies the relation Im = k lIm1,


1
then
(a) k= e
(b) l= m
(c) k =1
(d) None of
e
these
Let f be a positive function.
Let k
=

1k

f{x(1
x)}dx

1
x f{x(1 x)}dx
, isI2
2
=
(b) k

5.

6.

(a)
I1
(c) 1 / 2
when
2k1>
x
If f(t)dt = x+

f(t)dt,
0

(a) 1/2
(c) 1
1

1k
(d)
1

0. Then I / I2

then the value of is

f(1)

1
x

dx is equal
1 x to

[AMU 2000]

7.

1
3

(d)
3

(b)

2
3

If n is any integer,
then

edx

2
cosx

3
cos
(2n+1)x

[IIT 1985; RPET 1995; UPSEAT 2001]

The value of the definite


integral

(b) 1
(d) None of these

(a) x
(c) 0
8.

[IIT 1998; AMU 2005]

(b) 0
(d) 1/2

(a) 1
(c)

[IIT 1997 Cancelled]

(a) 0, 17

1
(c)
0, 27

x dx

x 3+ 16

(b) [0, 1]

(d) None of
these

lies in the interval [a, b]. The smallest such


interval is

9.

Let a,b,c be non-zero real numbers such


0 (1+ cos8 x)(ax2 +cosbx + c)dx=2
that
0
Then the quadratic equation ax 2 + bx + c= 0 has

(1+ 8 x)(ax2 + bx + c)dx

[IIT 1981; CEE 1993]

10.

11.

(a) No root in (0, 2)


(b) At least one root in (0,
2)
(c) A double root in (0, 2)
(d) None
x of these
If f(x)= | t| dt,
then
(a)
(b)
(c)
(d)

x1,

f and f are continous for


x+1>
0
f is continous but f is not continous for x+
1> 0

f and f are not continous at x = 0


f is continous at x= 0 but f is

not sox
Let g(x)= f(t)dt where21

3 g(2)<

cos 2x
4
dx, a> 0,
1+ a is

(b) a

(d) 2

If f(x)=

1+ e

f(a)

, I1 =

14.

(d) 2< g(2)<

(a)
(c)
13.

The value of

2005]

, then the value of


2
is
I

, and I

xg{x(1

f(a)x)}dx

[IIT Screening 2001; AIEEE

I1

f(a)

[AIEEE 2004]

=
g{x(1
x))}dx

(a) 1
(b) 3
2
f(a)
(c) 1
(d) 2
Let f : R R and g : R R be continuous functions, then the value of the
integral
/2

[f(x)+ f(x)] [g(x)


g(x)]dx =

[IIT 1990; DCE 2000; MP PET 2001]

/ 2

15.

[IIT Screening 2000]

(b) 0 g(2)< 2

and 0 f(t) 1 for t (1, 2],


then

f(t) 1,t[0,1]

(a) 1
2
2
(c) 3 < g(2) 5
12.

[MNR 1994]

(b) 1
(a)
(d) 0
(c) 1
The numbers P, Q and R for which the f(x)= Pe
Rx
function
log 4
39
are given by
[f(x) Rx]dx

2x

+ Qe x +satisfies the

f(0)=1,

conditions

f(log 2)= 31

and

2
=
(a) P = 2, Q =3, R= (b) P =5, Q= 2, R=
0

16.

(c) 4

(d) 3
P= 5, Q=6,
P=
5,
Q=2,
R=
3
10 2n

3
sin 27 x dx

2n1

n=1

sin

(a)
17.

18.

10

27

equals

2n+1

Given that

f(x)dx = a2,

2
(b) a
2
(d) a 2a+

(a) 0
(c) a2 1

(a)
60

[MP PET 2002]

x dx +
(b) 54

2n(d) 0
27 2
n=1
1
(c) 136
Let f(x)dx = 1, x f(x)dx = and
a

R=

[IIT 1990]

then the value of


0

x 2 dx

=
,
(x 2+a2 )(x +
2 b 2)(x +c 2) 2(a+ b)(b+ c)(c+
2
a)

(b)
20

(x a) f(x)dx =

is

x 2dx

then the value of


0

(x

+ 4)(x 2 + 9)

[Karnataka CET 1993]


40

(c)
19.

(d)
1

If l(m,n)=
0
1, n 1) is

80

(1+ t)n dt, then the expression for l(m,n) in terms of l(m+

[IIT Screening 2003]

2n
n

l(m+ 1,n
m+ 1 m+1)
1n
(b)
l(m+ 1,n1)
m+ 1

(a)

2n

n
+
l(m+ 1,n1)
m+ 1
m+
1m
(d)
l(m+ 1,n
n+ 1)
1
1+ 2 4 + 3 4 + ....+
1+ 2 3 + 3 3 + ....+
limn 3
=
lim n 4
n 5
n 5
n
n

(c)

20.

[AIEEE 2003]

1
(a)
30
(c) 1

If

t 2

= 2t ,
xf(x)dx
then f 5

(a)

2
5

1
5

t> 0, 4
=
25

(b)

(c) 2

[IIT Screening 2004]

5
2

(d) None of
these

2 and the line y= mx


For which of the following values of m, the area of the region bounded by the
curve 9
equals
y= x x

[IIT 1999]

(b) 2
(d) 4
(c)
2
Area enclosed between the 2 (2a x)=3 and line x = 2a above xx
curve y
axis is
3 a 2
(a) a 2
(b)
(a)

23.

(d)

21.

22.

(b) Zero

[MP PET 2001]

24.

(d) 3 a 2
(c) 2 a 2
What is the area bounded by the curves x 2 + y 2 = 9 and y 2 =
8 x is

2 2 + 9
(a) 0
(b)
1
3

25.

26.

[DCE 1999]

9 sin1

(c) 16
(d) None of
these
[IIT Screening
The area bounded by the curves y=| x | 1 and y =| x
2002]
| +1 is
(a) 1
(b) 2
(d) 4
(c) 2 2
The volume of spherical cap of height h cut off from a sphere of radius a is
equal
to
2
(a) h2(3a h)
(b) (a h)(2a
h2 ah)

[UPSEAT 2004]

(c) 4 h3
3

27.

(d) None of
these

3 / 2

If for a real number y, [y] is the greatest integer less than or equal to y, then the value
[2sin ofx]dx
the integral
/2
is
(a)

(b) 0

(c) 2

(d)

[IIT 1999]

28.

x +
If f(x)= A
sin
B, f = 1 f(x)dx =2A ,
1
0
then the constants A and B are
2 and
2
2
respectively
and
2
3
(a)
(b)
and
2

(c) 4 and 0
29.

If In = exx

then (b) 1 In

(a) In

ex
x n1dx
=
0 (d) In

(c) In

31.

tan

x dx , then limn[I
+ In2]
n

equals

(a) 1/2
(b) 1
(d) 0
(c)
The area bounded by the curves y= ln x , y= ln| x | , y=| ln x |isand y
=|
(a)ln|4 x
sq.|| unit
(b) 6 sq. unit
(c) 10 sq.
(d) None of
unit 1
these

32.

/4

0 n1dx,

30.

(d) 0 and

In =

[IIT 1995]

n+
x dx , (n
sin sin
x 1998]

N) equals

[AIEEE 2002]

[AIEEE 2002]

[Kurukshetra CEE

33.

(a) n

(b)

(c)

(d) 0

If

x 2 (x)dx

= then

0,

(a) 1<<
(c) 2
34.

35.

36.

37.

38.

100<

(2n+ 1)
2

< 1

| sin x |
dx

(b) < 0
(d) None of
these

is

(a) 20
(c) 10

[MNR 1994; Pb. CET 2001; UPSEAT 2000]

[AIEEE 2002]

(b) 8
(d) 18

2x(1+ sin x)
dx is
x
1+ cos 2
(a) 2 /4

(b)

(c) 0

(d) /2

[AIEEE 2002]
2

On the interval5 , 7

(6 cos t 2 sin t)dt

3 4
=
x
, the
f(x)=
(a) 3 3 + 2 2+ 1
(b) greatest value of the function
5 / 3
(c) Does not
3 32 21

1 x 3 (d) None
2 x 2 of
exist 1 x 2
If I1 = 2 dx, I2= 2 dx, I3= 2 dx , I
these
4
0
0
1
(a) I3 = I4
(b) I3 > I4
=
(c) I2 > I1
(d) I1 > I2

If 2 f(x)
3 f 1 = x 2,
f(x) dxis equal to
then
1
x

x 3

dx

, then

[AIEEE 2005]

3
(a) ln 2
5
(c) 3ln 2
5

3 (1+ ln
(b) 2)
5

(d) None of
these

[J & K 2005]

39.

40.

If

x
a

3dx

b
2
= 0 and x 2dx =
, then the value of a and b will be
a respectively
3

[AMU 2005]

(b) 1,1
(a) 1, 1
(c) 1,1
(d)
1,1
The sine and cosine curves
intersects infinitely many times giving bounded regions of equal areas. The area of
one of such
region is
[DCE 2005]
(a) 2
(b) 2 2
(c) 3 2
(d) 4 2

(Indefinite Integral )
1
6
11
16
21
26
31
36

b
b
a
c
a
b
c
b

2
7
12
17
22
27
32
37

b
d
b
c
a
c
c
b

3
8
13
18
23
28
33
38

a
a
a
c
a
a
a
a

4
9
14
19
24
29
34
39

a
c
a
a
d
a
d
d

5
10
15
20
25
30
35
40

d
d
a
a
a
d
c
a

5
10
15
20
25
30
35
40

a
a
d
d
b
b
b
b

Definite Integral
1
6
11
16
21
26
31
36

d
b
b
d
a
a
a
b

2
7
12
17
22
27
32
37

(d
c
c
a
b
c
c
d

3
8
13
18
23
28
33
38

b
a
d
a
b
c
c
b

89 of 89

4
9
14
19
24
29
34
39

)c

b
d
a
b
c
d
d

Вам также может понравиться